Several consumer protection agencies have filed suit, seeking to bar distributors from advertising treatments for baldness that brings no discernible improvement and may even result in potential harm.
that brings no discernible improvement and may even result in potential harm
that bring no discernible improvement and may even prove harmful
bringing no discernible improvement and even being harmful
that brings no discernible improvement and may even potentially result in harm being done
that bring no discernible improvement, maybe even resulting in harm
Above is a typical Sentence Correction question. In this chapter, we’ll look at how to apply the Kaplan Method to this question, discuss the grammar rules being tested, and go over the basic principles and strategies that you want to keep in mind on every Sentence Correction question. But before you move on, take a minute to think about what you see in this question and answer some questions about how you think it works:
What skills and concepts does this question test?
How does answer choice (A) relate to the given sentence?
What other patterns do you notice in the answer choices?
What GMAT Core Competencies are most essential to success on this question?
Many students, upon seeing Sentence Correction questions for the first time, think of them as “grammar” questions. It’s true that correct answers often fix grammar mistakes, but you don’t need to know the names of grammar rules or of different verb conjugations. What is it that you’re being asked to do on Sentence Correction questions? These questions ask you to pick the answer that most clearly, correctly, or effectively gets across the idea of the sentence.
Sentence Correction questions cover a range of grammatical errors, some of which are so obscure that even good writers commit them. However, you don’t have to be a grammar expert to do well on this section. All you need is a mode of attack and some knowledge about what does—and does not—constitute good GMAT grammar. Most of this chapter focuses on these common errors and how you can identify the answer choice that the GMAT considers correct.
Another key element in GMAT Sentence Correction is style, or what the directions for this question type call “effectiveness of expression.” That means English that is clear and exact and without awkwardness, ambiguity, or redundancy. (Note that it doesn’t have to be interesting. In fact, the test is set up to see whether you get worn down by difficult, often boring prose or whether you rise above that to stay involved—and awake.)
Each Sentence Correction sentence will contain an underlined portion and ask you which of the choices best fits in place of that underlined portion. Answer choice (A) will always repeat exactly that underlined part of the sentence. So, choice (A) is correct when there’s no error. Recognizing this pattern means that you never need to spend any time reading choice (A) once you’ve read the original sentence. Given the statistically random distribution of correct answers across the five choices, you can anticipate that choice (A)—the sentence is correct as written—will be correct approximately 20 percent of the time.
Spotting patterns in how the answer choices are presented is the bedrock of the Kaplan Method for Sentence Correction. As you scan through these answer choices, you’ll see that they tend to split easily into groups based on the varying ways they handle the grammatical issues in the sentence. Two of the choices start with that bring, two with that brings, and one with bringing.
We will return to analyze this particular question fully once we’ve learned the Kaplan Method, but for now just know that we call this phenomenon a “split”—certain patterns in how the answer choices are constructed allow us to place them into groups, sometimes even eliminating two or three answer choices solely on the basis of this categorization. Most Sentence Correction questions have a 3-2 or 2-2-1 split. This strategic, pattern-oriented approach will allow you to narrow the choices down to the one correct answer with the confidence and speed of an expert.
Pattern Recognition and Attention to the Right Detail are the Core Competencies central to Sentence Correction questions. The typical Sentence Correction question contains two or more errors. The testmakers reward you for being able to quickly spot and correct these problems that impair effective communication. Time is of the essence with these questions; the sentences vary in length and complexity, so you’ll have to move considerably faster on the short ones to leave time for the long ones. Knowing the main types of grammatical errors that show up repeatedly on Sentence Correction questions and being able to analyze the patterns in how the answer choices are presented will help you move through these questions efficiently and accurately.
Here are the main topics we’ll cover in this chapter:
Question Format and Structure
The Kaplan Method for Sentence Correction
Commonly Tested Grammar on the GMAT
The GMAT Verbal Reasoning section includes about 16 Sentence Correction questions, which are mixed in with Critical Reasoning and Reading Comprehension.
The directions for Sentence Correction questions look like this:
Directions: Each Sentence Correction question presents a sentence, part or all of which is underlined. Below each sentence you will find five ways to phrase the underlined portion. The first answer choice repeats the original version, while the other four choices are different. If the original seems best, choose the first answer choice. If not, choose one of the revisions.
In choosing an answer, follow the norms of standard written English: grammar, word choice, and sentence construction. Choose the answer that produces the most effective sentence, aiming to eliminate awkwardness, ambiguity, redundancy, and grammatical error.
Sentence Correction tests your command of standard written English—the rather formal language that is used in textbooks and scholarly periodicals. It’s the language that’s used to convey complex information precisely, as opposed to the casual language that we use for everyday communication. The good news is that you do not need to know every grammar rule for these questions. Errors reflecting certain rules show up repeatedly on the GMAT. Focus on mastering these commonly tested rules—that’s how to get the biggest bang for your study-time buck.
Now it’s time to learn how to bring together all of the strategies you read about above into a consistent protocol for approaching Sentence Correction questions. Kaplan has developed a Method for Sentence Correction that you can use to attack each and every Sentence Correction question. Through regular practice, this method will become second nature by Test Day.
The Kaplan Method for Sentence Correction |
1. Read the original sentence carefully, looking for errors. 2. Scan and group the answer choices. 3. Eliminate choices until only one remains. |
Read the sentence. Look for things that sound wrong but also keep your eyes peeled for signs of the classic errors that the GMAT loves to repeat. If you spot an error, eliminate (A) immediately. If you don’t spot an error the first time through, don’t bother rereading. You’re no more likely to spot a problem the second time around—especially because there may not be an error at all! Instead, move straight to Step 2.
Instead of wasting time reading each answer choice individually, quickly scan and compare the answers with one other. If you spotted an error in Step 1, sort the answer choices into two groups: those that do not fix the error (which you can eliminate) and those that appear to fix it.
If you didn’t spot an error, try to zero in on a grammatical or stylistic difference that splits the answer choices into distinct groups. This will let you identify one of the issues that the question is testing. Once you know what is being tested, you can apply your knowledge of English grammar to determine which group is correct—thereby eliminating multiple answers at once.
If more than one choice remains, go back to Step 2 and scan again to find another difference. Then eliminate accordingly. Repeat this process until only one answer remains.
Important Pacing Tip: If more than one choice remains after you have eliminated all of the answers that you are sure are wrong, just go with your best guess. If you don’t know the rule by Test Day, you probably won’t successfully teach it to yourself while taking the exam. You’ll get a much higher score by investing that time in other questions. If you are working on a quiz or a practice test, of course, reading the answer explanation closely will help you to learn the important rules so you can use them successfully on Test Day.
Now let’s apply the Kaplan Method to the Sentence Correction question you saw earlier:
Several consumer protection agencies have filed suit, seeking to bar distributors from advertising treatments for baldness that brings no discernible improvement and may even result in potential harm.
that brings no discernible improvement and may even result in potential harm
that bring no discernible improvement and may even prove harmful
bringing no discernible improvement and even being harmful
that brings no discernible improvement and may even potentially result in harm being done
that bring no discernible improvement, maybe even resulting in harm
The underlined phrase is a clause that is describing something in the first part of the sentence. (The use of the word that is a good clue that you are working with a clause.) What in the first part of the sentence “brings no discernible improvement” and may cause harm? Treatments. But you can’t say treatments brings.
You’ve found a problem with this sentence (subject-verb agreement), so (A), the original structure, can’t be correct. The correct answer will fix the subject-verb agreement problem without introducing other errors.
Now you’d quickly scan the choices, looking for any that repeat the error. That’s (D).
If you didn’t spot the error at first, you’d look for the main differences among the choices. Here, you have a 2-2-1 split: two say brings, two say bring, and one says bringing. Which is correct?
If you spotted the error, you eliminated (A) and (D) as wrong, which leaves bring and bringing.
If you didn’t spot the error, you’d now look at your groups of answer choices. Two answer choices say brings, two say bring—one is plural, the other singular. This is very likely about subject-verb agreement, one of the testmakers’ favorite types of errors. So what brings no improvement? Is it baldness? There’s a for before baldness, so baldness can’t be the subject. Plus “Baldness brings no discernible improvement” just doesn’t make sense. It has to be treatments. You’d eliminate (A) and (D) and turn your attention to bring/bringing.
You’d eliminate bringing, either because you recognize that the continuous tense is awkward and unnecessary here or because you know that on the GMAT, –ing forms are almost always wrong in verb questions. Plus, only one answer choice uses bringing, so eliminate it and move on.
Scanning between (B) and (E), you spot a difference at the end: and may even prove harmful versus maybe even resulting in harm. Prove is parallel with bring, and resulting is not. Eliminate (E). That leaves only one answer, (B).
TAKEAWAYS: THE KAPLAN METHOD FOR SENTENCE CORRECTION
If you don’t spot an error in the sentence, immediately scan the answer choices for differences. Focus on one error at a time. Usually you will find a 3-2 or 2-2-1 split somewhere in the wording.
Some differences will provide an easier basis by which to eliminate than others. If you see a difference but can’t determine which alternative is better, see if there are other differences that you can use.
Knowing how to spot the differences in the answer choices is only half the battle. Learning the testmakers’ favorite errors, so that you can figure out which version is correct, is the other half.
Answers and explanations at end of chapter
The rise in the number of new housing starts in the final two quarters of last year suggest that the sluggish economy should continue its recovery into the first quarter of this year, but weak job growth in the private sector continues to worry some economists.
suggest that the sluggish economy should continue its recovery into the first quarter of this year, but
suggest that the sluggish economy will continue to recover in the first quarter of this year, but that
suggest that the sluggish economy is continuing to recover in the first quarter of this year, but
suggests that the sluggish economy should continue its recovery into the first quarter of this year, but that
suggests that the sluggish economy will continue to recover in the first quarter of this year, but
The green flash, an atmospheric refractive phenomenon whereby the top edge of a setting sun will momentarily turn green, rarely is seen by the naked eye, primarily on account of requiring specific favorable conditions to occur.
on account of requiring
on account of their requiring
because they require
because it requires
because of requiring
A new study has found that college students had been more involved in on-campus political activities during the last few years than at any time in the past two decades.
had been more involved in on-campus political activities during the last few years than at any time
had been more involved in on-campus political activities during the last few years than at any other time
have been more involved in on-campus political activities during the last few years than at any other time
have been more involved in on-campus political activities during the last few years than at any time
are more involved in on-campus political activities during the last few years as compared to any other time
Depending on which scholar you consult, Christopher Columbus, Leif Ericson, or the Chinese eunuch Zheng Ho is credited with being the first explorer from the Eurasian continent to have traveled to the New World by ship.
is credited with being the first explorer from the Eurasian continent to have traveled to the New World by ship
is credited to be the first explorer from the Eurasian continent to have traveled to the New World by ship
is credited to have been the first explorer from the Eurasian continent to have traveled to the New World by ship
are credited with being the first explorers from the Eurasian continent to have traveled to the New World by ship
are credited to be the first explorers from the Eurasian continent to have traveled to the New World by ship
Noting that its revenues had fallen due to a recent prolonged slump in CD sales, the music-store chain announced that it would be forced to raise prices at all of its outlets.
its revenues had fallen due to a recent
its revenues have fallen due to a recently
its revenues are falling due to a recently
their revenues are falling due to a recent
their revenues had fallen due to a recent
Doing well on Sentence Correction questions begins with knowing how to approach them and then learning the errors that appear most frequently. Test takers who learn the most commonly tested patterns on GMAT Sentence Correction will be able to answer these questions confidently and efficiently. In this section, we’ll look at the seven most commonly tested areas of grammar and usage:
Verbs
Pronouns
Modification
Parallel Structure
Comparisons
Usage/Idioms
Miscellaneous Errors
It’s important to master these areas of grammar and usage, but keep in mind that many questions, particularly those of higher difficulty, will test multiple concepts. Many questions contain more than one error. Using the Kaplan Method for Sentence Correction will give you an efficient way to focus on one error at a time. With both knowledge of these commonly tested areas and mastery of the Kaplan Method, you’ll be able to handle all, even the toughest, Sentence Correction questions.
A sentence is defined as an independent sequence of words that contains a subject and a verb. Verbs must agree with their subjects. Singular subjects have singular verbs, and plural subjects have plural verbs. If you’re a native English speaker, this is probably so automatic that you may wonder why the GMAT tests it at all. But the testmakers craftily separate subject and verb with lots of text to make it harder to recognize whether the subject and verb agree. Also, it is sometimes hard to tell whether the subject is singular or plural. You should look out for the following common subject-verb agreement issues:
Long modifying phrases or clauses following the subject
Phrases and clauses in commas between the subject and the verb
Subjects joined by either/or and neither/nor
Sentences in which the verb precedes the subject
Collective nouns, such as majority, committee, audience, team, group, flock, family, especially when followed by a prepositional phrase containing a plural noun (“the group of legislators”):
Collective nouns take a singular verb when the members of the collective act as a unit (“the flock of geese is flying south”) or are non-countable (“a lot of water was spilled”).
When the members of the collective act as individuals, the collective noun takes a plural verb (“the majority of voters favor the proposal”; “a number of solutions are possible”).
Answers follow this exercise.
Correct each of the following subject-verb errors.
The depletion of natural resources, in addition to the rapid increase in utilization of these resources, have encouraged many nations to conserve energy.
There is, without a doubt, many good reasons to exercise.
Among the many problems plaguing suburbanites is the ubiquity of shopping malls, the increasing cost of gasoline, and the unavailability of mortgages.
The neighbors told police investigators that neither Annette nor her brother are capable of telling the truth.
The assembly of delegates intend to scrutinize the governor’s policy decisions.
Depletion is the subject. Correct by changing have to has or by changing in addition to to and.
There are many good reasons. A good strategy for checking subject-verb agreement is to ignore, temporarily, any parts of the sentence that are set off by commas.
If the sentence ended at malls, is would be correct. But because there is more than one problem listed, are is the correct verb here.
“Neither Annette nor her brother is capable.” In or/nor constructions, the verb agrees with the subject to which it is closest. So if it were “Neither Annette nor her friends,” are would be called for.
Even though delegates is plural, the subject of the sentence, assembly, is a singular noun referring collectively to the group. Assembly therefore takes a singular verb: “The assembly intends to scrutinize.”
A verb tense indicates the order in which separate actions or events occur. Deciding which verb tense is appropriate in a given situation isn’t just a matter of grammar; it’s also a question of logic. Many GMAT sentences are long and complicated, involving or implying several different actions. The correct tenses make the sequence of events clear.
To determine whether the verbs in a sentence are in the proper tenses, pick one event as a standard and measure every other event against it. Ask yourself whether the other events are supposed to have happened before the standard event took place, after it took place, or while it took place. Those aren’t mutually exclusive options, by the way: it is possible in English to have one action start before a second action and continue during that second action.
A frequent GMAT verb error is the inappropriate use of –ing forms: “I am going, I was going, I had been going,” and so on. As far as the GMAT is concerned, the only reason to use an –ing form is to emphasize that an action is continuing or that two actions are occurring simultaneously. To remember this rule, think of the word during and its –ing ending. Other than that, pick a simpler tense—one that doesn’t use the –ing form. In other words, avoid –ing forms as much as possible.
Simple Present—I am—Used for statements of fact, and sometimes to describe an action happening now, with no contextual information about when it started.
Simple Past—I was—Used for an action that happened at a specified time in the past.
Simple Future—I will—Used for an action that will happen in the future.
Past Perfect—I had been—Used for an action that happened before another past action (e.g., “I had been on the subway for 30 minutes before I realized that I was going the wrong direction.”).
Present Perfect—I have been—Used for an action that started in the past but is still continuing now (e.g., “I have been on the subway for two hours now, and I still don’t know where I’m going!”) or for past events that happened at an unspecified time (e.g. “He has read Don Quixote seven times.”).
There are other verb tenses in the English language, but these are the ones that are tested most often on the GMAT.
Answers follow this exercise.
Correct the verb tenses in each sentence.
The criminal escaped from custody and is believed to flee the country.
Some archaeologists believe that the Minoans of 3,700 years ago had practiced a religion that involved human sacrifice.
If the experiment works, it will be representing a quantum leap forward for pharmaceutical chemistry.
He had seen that movie recently, so he doesn’t want to see it tonight.
By the time she retires, she will save enough money to allow her to live comfortably.
She already closed the door behind her when it occurred to her that she wasn’t able to get back in later.
The criminal escaped correctly uses simple past tense to refer to an event that happened at a specific time. The believing happens now, so simple present is believed is correct. However, the fleeing happened at an unknown and unspecified time in the past, so the present perfect should be used: “… is believed to have fled the country.”
Here, there’s no indication the Minoans practiced human sacrifice for a while and then did something else. So use the simple past practiced instead of the past perfect had practiced.
The experiment won’t be representing a quantum leap; it will represent a quantum leap.
Had plus a past tense verb is used to indicate which of two things that went on in the past occurred earlier. That’s not necessary in this sentence. “He saw that movie recently, so he doesn’t want to see it tonight.” (“He had seen the movie recently, so he didn’t want to see it tonight,” also works, although it changes the meaning of the sentence to indicate that the desire happened earlier tonight instead of happening now.)
Here, we’re indicating an action that began in the past but will end in the future. Think of it this way: at some future time, what will have happened? “… she will have saved enough money.”
Closed, occurred, and wasn’t able to get back in are all in the simple past tense. But you need to indicate that she first closed the door and then something occurred to her—namely, that she wouldn’t be able to do something in the future. “She had already closed the door behind her when it occurred to her that she wouldn’t be able to get back in later.”
Now let’s use the Kaplan Method on a Sentence Correction question dealing with verbs:
The governor’s approval ratings has been extremely high until a series of corruption scandals rocked his administration last year.
has been extremely high until
have been extremely high until
had been extremely high until
were extremely high as
had been extremely high as
Whenever a sentence contains an underlined verb, you need to make sure that it agrees with its subject and is in the correct tense. Here, the underlined portion contains a singular verb, “has been,” that disagrees with the plural subject, “ratings.” The verb is also is in the wrong tense—“has been” indicates that the ratings are still high, but the sentence contradicts that. You can eliminate answer choice (A) immediately.
Now it’s time to look for a split in the answer choices. You see that the choices begin with many different verb forms: two “had been,” one “were,” one “have been,” and one “has been.” That’s not a very helpful split. If you don’t find a split at the beginnings of the answer choices, look for a split at the ends. Answer choices (A), (B), and (C) end with “until,” whereas (D) and (E) end with “as”; a 3-2 split.
You eliminated answer choice (A) because the subject, “ratings,” is plural, so the singular verb, “has been,” cannot be correct. But verb tense is also at issue here. The correct verb tense is “had been extremely high,” because the past perfect tense is used to indicate that something had already happened in the past before something else happened in the past. Here, the governor’s ratings had been high, until scandals rocked his administration. This eliminates (B) and (D). And (E) can be eliminated, since changing the preposition from “until” to “as” loses the sense that the scandals occurred before, and led to, the reversal in the governor’s approval ratings. For the record, note that (D) also contains this error. The GMAT will often give you multiple opportunities to eliminate answer choices. This leaves (C) as the only flawless answer.
TAKEAWAYS: VERBS
A complete sentence consists, at minimum, of a subject and a verb.
The verb must agree with the subject of the sentence; plural subjects take plural verbs and singular subjects take singular verbs.
The verb tense must match the meaning of the sentence as a whole.
Don’t fall for needlessly complicated verb tenses. Go for the simplest verb tense that makes sense given the time frame of the sentence.
Use the past perfect tense (had done, had seen) to indicate something that happened prior to another past event.
Use the present perfect tense for an action that happened at an unspecified time in the past (have read, has said) or an action that started in the past and has continued until the present time (have lived, has been, have had).
Use the present perfect continuous tense (have been studying, has been waiting) for an action that started in the past and is still continuing now.
Answers and explanations at end of chapter
After all the research that has been conducted in the last 30 years, it is readily apparent that there are, without question, a wealth of good reasons to do aerobic exercise regularly.
there are, without question, a wealth of good reasons to do aerobic exercise regularly
there are, without question, a wealth of good reasons to exercise aerobically regularly
there are, without question, a wealth of good reasons to engage regularly in aerobic exercise
there is, without question, a wealth of good reasons to engage regularly in aerobic exercise
there is, without question, a wealth of good reasons to do regular aerobic exercises
As state governments become less and less able to support higher education in the coming years, universities have been becoming more and more dependent on alumni networks, corporate sponsorships, and philanthropists.
become less and less able to support higher education in the coming years, universities have been becoming
are becoming less and less able to support higher education in the coming years, universities have become
become less and less able to support higher education in the coming years, universities will become
become less and less able to support higher education in the coming years, universities have become
are becoming less and less able to support higher education in the coming years, universities will become
The most common breed of rabbit, Oryctolagus cuniculus, is native to Europe but was accidentally introduced into the Australian continent in the 1850s, causing decades of habitat destruction and crop devastation before its numbers were finally brought under control by bounty hunters.
is native to Europe but was accidentally introduced into the Australian continent in the 1850s, causing
is a native of Europe but was accidentally introduced into the Australian continent in the 1850s, which caused
was a native of Europe but was accidentally introduced into the Australian continent in the 1850s, causing
had been native to Europe but was accidentally introduced into the Australian continent in the 1850s, which caused
had been a native of Europe but was accidentally introduced into the Australian continent in the 1850s, causing
A recent spate of news reports questioning the long-term health benefits of high-fat diets have done little to convince its practitioners that they should follow more traditional weight-loss plans.
have done little to convince its practitioners that they should follow more traditional weight-loss plans
have done little to convince their practitioners to follow more traditional weight-loss plans
has done little to convince its practitioners to follow more traditional weight-loss plans
has done little to convince practitioners of these diets to follow more traditional weight-loss plans
has done little to convince practitioners of these diets they should follow more traditional weight-loss plans
Answers and explanations at end of chapter
Some economists contend that the way markets react to financial news are based as much on irrational fears and expectations than on sound fiscal analyses.
are based as much on irrational fears and expectations than on
are based as much on their irrational fears and expectations as they are on
is based as much on irrational fears and expectations than on
is based as much on irrational fears and expectations as on
has been based on irrational fears and expectations as much as
The newspaper reported that some homes that were destroyed and severely damaged in the hurricane last year were built too close to the coastline.
some homes that were destroyed and severely damaged in the hurricane last year were
some homes that were destroyed or severely damaged in the hurricane last year had been
some homes the hurricane destroyed and severely damaged last year have been
last year the hurricane destroyed or severely damaged some homes that have been
last year some of the homes that were destroyed or severely damaged in the hurricane have been
The activism of state citizens, who have demanded safer road conditions as well as stiffer penalties for intoxicated drivers, have led to a significant decrease in the number of traffic accidents.
have led to a significant decrease in the number
have led to significant decreases in the amount
has led to a significant decrease in the number
has been significant in the decrease in the number
has significantly decreased the amount
Pronoun errors are one of the most common Sentence Correction issues on the GMAT. Luckily, the GMAT doesn’t test every kind of pronoun error. Common errors fall into two categories: reference and agreement. Pronoun reference errors mean that a given pronoun does not refer to—or stand for—a specific noun or pronoun in the sentence (its antecedent). The pronouns that cause the most trouble on the GMAT are it, its, they, their, them, which, and that.
For pronoun agreement errors, it’s a question of numbers: perhaps a pronoun that refers to a singular noun is not in singular form, or a pronoun that refers to a plural noun is not in plural form.
As usual, the GMAT presents camouflaged examples of these two mistakes. Whenever you see a pronoun in the underlined portion of a sentence, look out for the following:
Pronouns, such as it and they, that are often misused on the GMAT (and in everyday life)
Pronouns that don’t clearly refer to a specific noun
Pronouns that don’t agree in number or gender with their antecedents
Answers follow this exercise.
Correct the following common pronoun reference errors.
Beatrix Potter’s stories depict animals in an unsentimental and humorous manner, and she illustrated them with delicate watercolor paintings.
There is no known cure for certain forms of hepatitis; they hope, though, that a cure will be found soon.
If the partners cannot resolve their differences, the courts may have to do it.
In order to boost their name recognition, the Green Party sent canvassers to a busy shopping mall.
It is now recognized that the dangers of nuclear war are much graver than that of conventional warfare.
One of the men complained about the noise in the hallway, and they don’t want to identify themselves.
She is clearly intended to refer to Beatrix Potter, but notice that the proper noun Beatrix Potter doesn’t appear anywhere in this sentence; a pronoun cannot refer to a modifier, even a possessive modifier such as Beatrix Potter’s. There’s a second problem as well: it’s not clear whether the them that are illustrated are the stories or the animals. Here’s a rewrite that solves all the problems: “Beatrix Potter not only wrote stories that depicted animals in an unsentimental and humorous manner, but also illustrated each story with delicate watercolor paintings.”
It’s unclear what they refers to. The only plural noun is forms, but it can’t be the forms of hepatitis that are hoping for a cure. It must be scientists or some other group of people: “scientists hope to find a cure soon.”
It is the unclear pronoun here. There’s no singular noun in the sentence for it to refer to. The main clause should read: “… the courts may have to do so.”
A pronoun or possessive should match the form of the noun it refers to. Use its and not their in place of the Green Party, because party, like audience, is a singular noun that stands for a collective group: “In order to boost its name recognition, the Green Party …”
Dangers is plural, so the pronoun “that” should be plural as well: “… than those of conventional warfare.”
The antecedent of “they” and “themselves” is “one,” so the pronouns do not agree in number or gender with their antecedent. Because the gender of “one” is male (“one of the men”), the pronouns should be changed to “he” and “himself,” and the verb “don’t” should be changed to the singular “doesn’t.”
Now let’s use the Kaplan Method on a Sentence Correction question dealing with pronouns:
Despite the platform of the opposition party supporting the measure, they keep voting against campaign finance reform in Congress.
the platform of the opposition party supporting the measure, they keep
the opposition party’s platform supporting the measure, they keep
the opposition party’s platform which supports the measure, it keeps
support of the measure being in the opposition party’s platform, it keeps
the opposition party’s platform supporting the measure, party members keep
Pattern Recognition and Attention to the Right Detail are essential to spotting pronoun issues in Sentence Correction. Here, you should be paying attention to the word “they” in the underlined portion of the sentence. Colloquially, it’s common to use “they” as a nebulous pronoun with no clear antecedent. On the GMAT, though, such usage is always wrong. In this sentence, it isn’t clear who “they” refers to, so you know that’s an issue that needs to be fixed by the correct answer.
The beginnings of the answer choices don’t yield much in the way of splits, but the ends definitely do: (A) and (B) end with “they,” (C) and (D) end with “it,” and (E) dispenses with pronouns entirely. You have a 2-2-1 split.
The pronoun use here is wrong, because the sentence does not contain an antecedent plural noun to which “they” could refer. So, (A) and (B) are incorrect. (C) and (D) contain the singular pronoun “it”—but once again, the pronoun reference is confusing and wrong. It’s not the party platform that keeps voting against the measure, as these choices imply. Both (C) and (D) can be eliminated. Only choice (E), which avoids faulty pronoun reference and makes it clear that party members keep voting against the measure, makes sense and is the correct answer.
TAKEAWAYS: PRONOUNS
A pronoun is a word that stands for a noun.
A pronoun must refer unambiguously to a specific noun, known as its antecedent.
A pronoun must agree in number and gender with the noun it replaces.
When you see a pronoun underlined in the original sentence, focus on that pronoun and determine whether it is being used correctly.
The pronouns that cause the most trouble on the GMAT are it, they, its, their, them, which, and that.
Never use they to refer to a third-person singular noun on the GMAT, even if you don’t know or don’t want to specify gender.
Answers and explanations at end of chapter
The government has imposed sanctions and restricted foreign aid to a renegade nation last month after it violated the terms of a worldwide arms control treaty.
has imposed sanctions and restricted foreign aid to a renegade nation last month after it violated
has imposed sanctions on and restricted foreign aid to a renegade nation last month after it violated
imposed sanctions on and restricted foreign aid to a renegade nation last month after that nation violated
imposed sanctions and restricted foreign aid to a renegade nation last month after violating
had imposed sanctions and restricted foreign aid to a renegade nation last month after it had violated
Every one of the police reports have been filed so that they can be located by their case number, the date they were created, or the victim’s last name.
have been filed so that they can be located by their case number, the date they were created, or the victim’s last name
have been filed so that they can be located by their case numbers, the dates they were created, or by the victims’ last names
has been filed so that they can be located by their case number, the date they were created, or the victim’s last name
has been filed so that it can be located by its case number, the date it was created, or the victim’s last name
has been filed so that it can be located by its case number, the date it was created, or by the victim’s last name
Although swimming can cause cramps in a swimmer’s legs or feet, they can usually be avoided if the swimmer avoids overexertion and performs stretching exercises before entering the pool.
Although swimming can cause cramps in a swimmer’s legs or feet, they
Cramps in a swimmer’s legs or feet caused by swimming
Swimming can cause cramps in a swimmer’s legs or feet, which
The fact that swimming can cause cramps in a swimmer’s legs or feet
Swimming can cause cramps in a swimmer’s legs or feet, although they
Some mathematicians argue that to permit a candidate to win an election because they have won a plurality vote is like ranking a student who earned three As and two Fs higher than one who got two As and three Bs.
to permit a candidate to win an election because they have won a plurality vote is like ranking a student
permitting a candidate to win an election because they have won a plurality of votes is like ranking a student
permitting candidates to win elections because they have won a plurality of votes is like to rank a student
permitting candidates to win elections because they have won a plurality of votes is like ranking a student
to permit candidates to win elections because they have won a plurality of votes is like ranking students
Answers and explanations at end of chapter
According to a recently enacted law, any organization that engages in lobbying activities is required either to notify their members about the percentage of dues that are used for lobbying activities or pay a proxy tax.
their members about the percentage of dues that are used for lobbying activities or
their members about the percentage of dues they use for lobbying activities or to
their members about the percentage of dues that are used for lobbying activities or they should
its members about the percentage of dues that are used for lobbying activities or it should
its members about the percentage of dues that is used for lobbying activities or to
The chief executive officer met with the board of directors to consider the possibility of a hostile takeover attempt by a competitor and how they would have to respond to deal with them.
how they would have to respond to deal with them
how to deal with them if action would become necessary
what action would be required for dealing with such an event
what action would be necessary to deal with such an event
the necessity of taking action in order to deal with it
Arguing that the dominance of an economic theory has more to do with the persuasive skills of its expositors than with its accuracy in predicting economic events, Professor McCloskey examines the rhetorical practices of economists.
its expositors than with its
these expositors than its
its expositors compared to its
the theory’s expositors than their
its expositors than with their
A modifier is a word, phrase, or clause that describes another part of the sentence. It should be placed as close as possible to whatever it is modifying. Adjectives modify nouns; adverbs modify verbs, adjectives, or other adverbs. Modifiers describe the word that they are right next to. (The only exception is the case of two modifiers; one has to be first.) The GMAT often creates modification errors by making a modifier appear to describe a word that it actually doesn’t. Use Pattern Recognition to help you spot modification errors on the GMAT.
The most common GMAT modification error is a long modifier at the beginning of the sentence. It should modify the subject of the sentence but will likely not do so properly. Another common modification error is a long modifier that appears in the middle or at the end of a sentence. Often such a modifier on the GMAT will logically describe something elsewhere in the sentence, but grammatically a modifying phrase must modify the word that comes immediately before or after it. The result of a misplaced modifier is a nonsensical sentence. Also look out for the following:
Sentences beginning or ending with descriptive phrases
That/which clauses, especially ones that come at the end of sentences
Answers follow this exercise.
In each of the following sentences, first identify what each clause or phrase is modifying. Then, fix each error you find.
Upon landing at the airport, the hotel sent a limousine to pick us up.
Based on the most current data available, the company made plans to diversify its holdings.
Small and taciturn, Joan Didion’s presence often goes unnoticed by those she will later write about.
I took several lessons to learn how to play tennis without getting the ball over the net even once.
The house overlooked the lake, which was set back from the shore.
The sentence seems to be saying that the hotel landed at the airport. Your common sense will tell you that upon landing at the airport really intends to modify the unnamed we, instead. So you could say, “Upon landing at the airport, we were met by a limousine the hotel had sent.”
As written, it sounds as though the company was based on current data. Based on the most current data available modifies the subject company. Obviously, though, what was based on the data were the plans, not the company. “Based on the most current data available, plans were made to diversify the company’s holdings.”
It’s Joan Didion—not her presence—who is small and taciturn. “Small and taciturn, Joan Didion often goes unnoticed …”
As written, without getting the ball over the net is describing how to play tennis. The intended meaning is much more likely to be “I took many tennis lessons before I could get the ball over the net even once.”
The misplaced modifying clause produces an absurd image: a lake that’s set back from its own shore. Of course, the which clause should follow the house: “The house, which was set back from the shore, overlooked the lake.”
Now let’s use the Kaplan Method on a Sentence Correction question dealing with modification:
Subjects tend to be vividly but disturbingly portrayed in Egon Schiele’s portraits, often his closest friends and relatives.
Subjects tend to be vividly but disturbingly portrayed in Egon Schiele’s portraits, often his closest friends and relatives
Subjects tend to be vividly but disturbingly portrayed in Egon Schiele’s portraits, who were often his closest friends and relatives
Subjects of Egon Schiele’s portraits, often his closest friends and relatives, tend to be vividly but disturbingly portrayed
In Egon Schiele’s portraits, the subjects, often his closest friends and relatives, tend to be vividly but disturbingly portrayed
Vividly but disturbingly, the subjects portrayed in Egon Schiele’s portraits tended to be his closest friends and relatives
The entire sentence is underlined, but don’t let that intimidate you. Use the Kaplan Method as you normally would and look for common errors. In this case, the error is a misplaced modifier at the end of the sentence. (Keep in mind that misplaced modifiers can occur anywhere in a sentence, not just at the beginning.) Here, the modifier is the final phrase set off by a comma, “often his closest friends and relatives.” This phrase should refer to “subjects,” but it’s placed right next to “portraits.” Always ask yourself when you see a modifier, “What should this word or phrase refer to? Is it as close to that word or phrase as possible?”
As you scan, be on the lookout for where the modifier is placed. In these answer choices, the modifying phrase appears in several different positions. Any choice placing it far from the word it modifies—“subjects”—should be eliminated. In (B), the modifying clause, “who were often his closest friends and relatives,” seems to refer to portraits rather than subjects. Choice (B) retains the same problem as (A), so you can eliminate it. Choice (D) places the modifier immediately after “subjects,” so this is likely to be your answer.
In choice (C), “subjects of Egon Schiele’s portraits” seems to be one syntactical unit, so the phrase “often his closest friends and relatives” appears correctly to modify subjects, even though it is not directly adjacent to subjects. However, (C) is wrong because it is unclear whether “tend to be vividly but disturbingly portrayed” refers to how these subjects are displayed in the portraits, or elsewhere; perhaps biographers of Schiele depict them in this manner.
Finally, choice (E) is incorrect because the adverbial phrase “Vividly but disturbingly” appears to refer to the verb “tended” rather than to the adjective “portrayed,” making it seem as if the subjects’ tendency to be Egon’s friends is what’s vivid and disturbing. Only choice (D) properly addresses this misplaced modifier problem; it is therefore your correct answer. Moreover, choice (D), unlike (C), makes it clear that the vivid but disturbing portrayal is in the portaits themselves.
TAKEAWAYS: MODIFICATION
Adjectives modify nouns; adverbs modify verbs, adjectives, or other adverbs.
A modifying phrase must clearly refer to what it modifies. It should be placed as close as possible to what it modifies.
Many GMAT modification errors involve misplaced modifiers at the beginning of the sentence. When there is a modifying phrase at the beginning of the sentence, make sure that the subject of that phrase is what follows the comma.
Misplaced modifiers can also occur at the middle and end of a sentence, and they may or may not be set off by commas.
When a particular modifying word or phrase appears in many different positions in the answer choices, determine exactly what it is meant to modify and pick the choice that places it where it belongs.
Answers and explanations at end of chapter
In an effort to determine whether changes in diet and exercise can reverse coronary heart disease, 1,800 elderly heart patients will participate in a Medicare program costing $12,960,000 over the next three years to reduce lifestyle-related risk factors.
1,800 elderly heart patients will participate in a Medicare program costing $12,960,000 over the next three years to reduce lifestyle-related risk factors
$12,960,000 will be spent by Medicare over the next three years in a program to reduce lifestyle-related risk factors in 1,800 elderly patients
over the next three years $12,960,000 will be spent on 1,800 patients by Medicare on a program to reduce lifestyle-related risk factors
Medicare is to spend $12,960,000 over the next three years on a program for reducing lifestyle-related risk factors in 1,800 elderly heart patients
Medicare will spend $12,960,000 over the next three years on a program to reduce lifestyle-related risk factors in 1,800 elderly heart patients
Children often pick up on gender norms very early in life, as many parents discover when their toddlers recoil at toys that have only been designed to appeal to toddlers of the opposite sex.
their toddlers recoil at toys that have only been designed to appeal to toddlers of the opposite sex
only their toddlers recoil at toys that have been designed to appeal to toddlers of the opposite sex
their toddlers recoil at toys that have been designed to appeal to toddlers of the opposite sex only
toys that have only been designed to appeal to toddlers of the opposite sex cause their toddlers to recoil
only toys that have been designed to appeal to toddlers of the opposite sex cause their toddlers to recoil
Based on the candidates’ performances in the televised debates, many pundits predicted that the rival was likely to score an upset defeat over the incumbent senator.
Based on the candidates’ performances in the televised debates
Basing it on the candidates’ performances in the televised debates
Basing their assessments on the candidates’ performances in the televised debates
With the candidates’ performances in the televised debates used as a basis
By the assessments of the candidates’ performances in the televised debates that they made
Hoping to avert another situation in which the candidate who won the popular vote could lose the electoral vote, it was proposed by Congress that the Electoral College be abolished.
it was proposed by Congress
there was a proposal by Congress
a proposal was made by Congress
Congress proposed
Congress will have proposed
Answers and explanations at end of chapter
Cheered by better-than-expected opening week box office returns, it was decided by the movie’s producer to give bonuses to the entire cast.
it was decided by the movie’s producer to give bonuses to the entire cast
the entire cast was given bonuses by the movie’s producer
the movie’s producer decided to give the entire cast bonuses
the decision of the movie’s producer was to give the entire cast bonuses
bonuses to the entire cast were given by the movie’s producer
States now have an incentive to lower the blood alcohol level that constitutes drunk driving by a federal law that withholds highway funds from those states that don’t enforce the applicable standard.
that constitutes drunk driving by a federal law that withholds
that constitutes drunk driving, because a federal law withholds
that constitutes drunk driving, because a federal law withheld
which constitutes drunk driving by a federal law that withholds
which constitutes drunk driving, because a federal law withholds
Using a seismic survey, hydrocarbons can be located even though they are buried far beneath Earth’s surface.
Using a seismic survey, hydrocarbons can be located even though they are buried far beneath Earth’s surface
Hydrocarbons can be located even though they are buried far beneath Earth’s surface, using a seismic survey
Locating hydrocarbons that are buried far beneath Earth’s surface, a geophysicist can use a seismic survey
Buried far beneath Earth’s surface, hydrocarbons can be detected using a seismic survey
Using a seismic survey, a geophysicist can locate hydrocarbons even though they are buried far beneath Earth’s surface
The basic concept behind parallelism is pretty simple: ideas with the same importance and function—nouns, verbs, phrases, or whatever—should be expressed in the same grammatical form. There are two types of constructions that test parallel structure on the GMAT:
Lists of items or a series of events
Two-part constructions such as from X to Y, both X and Y, either X or Y, prefer X to Y, not only X but also Y, and as much X as Y
Prepositions, articles, and auxiliaries can begin a list without needing to be repeated throughout. But if they are repeated, they must be in every element of the list.
Correct: “Will you travel by plane, car, or boat?”
Also correct: “Will you travel by plane, by car, or by boat?”
Incorrect: “Will you travel by plane, car, or by boat?”
Analogies, similes, and other comparisons all require parallel structure.
He was as brazen as his brother was diffident.
Seeing her smile was like feeling the warmth of the sun.
Answers follow this exercise.
For each of the following sentences, put parallel items into the same form.
The city’s decay stems from governmental mismanagement, increasing unemployment, and many businesses are relocating.
Tourists’ images of France range from cosmopolitan to the pastoral.
Excited about visiting New York, Jasmine minded neither riding the subways nor to cope with the crowded sidewalks.
To visualize success is not the same as achieving it.
I remember my aunt making her own dandelion wine and that she played the fiddle.
In my favorite Armenian restaurant, the menu is fascinating and the entrées exquisite.
Many businesses are relocating should be written as business relocation to parallel governmental mismanagement and increasing unemployment.
If you say the pastoral, you have to say the cosmopolitan. Or you could say “… from cosmopolitan to pastoral.”
To parallel riding, you need coping—not to cope.
To visualize should be visualizing to parallel achieving. Alternately, achieving could be written as to achieve to parallel to visualize.
Change my aunt making to that my aunt made to be parallel with that she played. Another possibility is to change that she played to playing to match making.
The menu is singular, but entrées is plural, so you must say are exquisite to parallel the phrase is fascinating.
Now let’s use the Kaplan Method on a Sentence Correction question dealing with parallelism:
Pablo Picasso’s genius is only fully revealed when one considers the various facets of his work as they developed through many artistic phases, beginning with his Red period, continuing through his Blue period, and finishing with his period of Cubism.
period, continuing through his Blue period, and finishing with his period of Cubism
period, continuing through his Blue period, and finishing with his Cubist period
period, and continuing through his Blue period and his Cubist period
period phase, his Blue period phase, and his phase of Cubism
period, his Blue period, and his period of Cubism
The original sentence contains a list, so your Pattern Recognition skills should tell you to check for parallel structure. Because the first two items in the list contain “Red period” and “Blue period,” you should expect the third item in the list to follow the adjective-noun pattern and contain “Cubist period.” Instead, this sentence ends with “period of Cubism,” breaking the pattern and making choice (A) incorrect.
Choices (B) and (C) end with “his Cubist period,” while (A) and (E) reference “his period of Cubism” and (D) ends with the similar construction “his phase of Cubism.” You have a 3-2 split.
You’ve established that (A) is incorrect, so eliminate (D) and (E) as well. This leaves only (B) and (C) with the correct parallel construction “cubist period.” The list should also contain the parallel phrases, “beginning … continuing … and finishing.” (C) is wrong because it drops “finishing,” which alters the meaning and also destroys the parallel structure. Only (B) exhibits parallel structure throughout the list and is the correct answer.
TAKEAWAYS: PARALLELISM
Items in a list must have parallel form.
Many two-part constructions set up parallel elements. Examples: “not only A but also B,” “from A to B,” and “either A or B.”
The key to handling parallel structure questions is consistency.
Analogies, metaphors, similes, and other comparisons all require parallel structure.
Answers and explanations at end of chapter
The creation of an independent treasury, establishing lower tariffs, and purchasing the Oregon Territory, all credited to the presidency of James Knox Polk, are among the significant accomplishments that persuade historians to rank this former governor of Tennessee as an above-average president.
The creation of an independent treasury, establishing lower tariffs, and purchasing
The creation of an independent treasury, establishing lower tariffs, as well as purchasing
The creation of an independent treasury, the establishment of lower tariffs, and the purchase of
Creating an independent treasury, the establishment of lower tariffs, and purchasing
Creating an independent treasury, the establishing of lower tariffs, and the purchasing of
The rapid-rail lines that were recently introduced in southern California have had positive effects: not only have they lowered pollution levels, they have helped in decongesting the region’s freeway system, which had been operating far above its capacity.
not only have they lowered pollution levels, they have helped in decongesting
the lines have not only lowered pollution levels, but also helped to decongest
not only have pollution levels been lowered, but they have helped to decongest
lines have lowered pollution levels, but also helped in decongesting
not only have they lowered pollution levels, but also have they helped decongest
A national study evaluating the need for gun control is currently attempting to document where criminals purchase guns, how gun owners are trained in the use of firearms, and what character traits are common to persons who are liable to turn to violence to resolve problems.
what character traits are common to persons who are liable to turn
what are the character traits common to persons who are liable to turning
the common character traits of persons who are liable to turn
the traits of character common to persons who are liable to turn
the traits of character that are common to persons who are liable to turn
Hearing Kenneth Branagh deliver the “St. Crispin’s Day” speech in Henry V is to be mesmerized by a great performer, with seemingly boundless emotional energy.
Hearing Kenneth Branagh deliver the “St. Crispin’s Day” speech in Henry V is to be mesmerized by a great performer, with seemingly
Hearing Kenneth Branagh deliver the “St. Crispin’s Day” speech in Henry V is to be mesmerized by a great performer, one with seeming
Hearing Kenneth Branagh deliver the “St. Crispin’s Day” speech in Henry V is being mesmerized by a great performer, one with seeming
To hear Kenneth Branagh deliver the “St. Crispin’s Day” speech in Henry V is being mesmerized by a great performer, with seemingly
To hear Kenneth Branagh deliver the “St. Crispin’s Day” speech in Henry V is to be mesmerized by a great performer, one with seemingly
Answers and explanations at end of chapter
By the time they completed their journey, the young explorers had overcome their fears, sharpened their survival skills, and had developed a healthy respect for nature’s potential destructiveness.
skills, and had developed a healthy respect
skills, and developed a healthy respect
skills and a healthy respect developed
skills, developing a healthy respect
skills, all the while developing a healthy respect
However much Americans may agree that the financing of elections with special interest money undermines democracy and that campaign finance reform would produce better government, it has been very difficult to push such measures through a Congress that has been elected using the old financing system.
However much Americans may agree that
Despite agreement among Americans to the fact
Although Americans agree
Even though Americans may agree
There is agreement among Americans that
South Pacific Vacations has a package tour to Sydney, Australia, for $999 per person, including airfare from Los Angeles, spending five nights at the Mega Hotel, and to take a harbor cruise, and round-trip airfare from New York is an additional $400.
spending five nights at the Mega Hotel, and to take a harbor cruise, and round-trip airfare from New York is
spending five nights at the Mega Hotel, and taking a harbor cruise; and in addition round-trip airfare from New York is
five nights at the Mega Hotel, and a harbor cruise; round-trip airfare from New York is
with five nights at the Mega Hotel, and a harbor cruise, with round-trip airfare from New York being
to spend five nights at the Mega Hotel, and to take a harbor cruise; round-trip airfare from New York is
Faulty comparisons account for a significant number of errors in GMAT Sentence Correction questions. Most relate to the very simple idea that you can’t compare apples to oranges. Of course, you want to compare things that are grammatically similar, but you also want to compare things that are logically similar. You can’t logically compare, say, a person to a quality or an item to a group. You have to compare one individual to another, one quality to another, one group to another.
Use Attention to the Right Detail and Pattern Recognition to keep an eye out for the following:
Key comparison words such as like, as, compared to, less than, more than, other, that of, and those of
Long modifying phrases between compared elements. Don’t be distracted! Ignore the modifying phrase at first so you can see the compared elements clearly.
Compared items must be parallel in terms of construction. If a preposition, article, or auxiliary is in one compared item, it needs to be in the other:
Incorrect: I would do anything for my mother but not my boss.
Correct: I would do anything for my mother but not for my boss.
Answers follow this exercise.
Fix the comparisons in the sentences below.
Like a black bear that I once saw in the Buenos Aires Zoo, the Central Park Zoo polar bear’s personality strikes me as being sadly neurotic.
The article questioned the popularity of jazz compared to classical music.
The challenger weighed 20 pounds less than that of the defender.
The Boston office contributes less to total national sales than any other U.S. branch.
The host paid more attention to his celebrity guest than the others.
Like creates a comparison, and you can compare only similar things. Here, you have to compare bears to bears or personalities to personalities. “Like a black bear I once saw in the Buenos Aires Zoo, the Central Park Zoo polar bear strikes me as being sadly neurotic.”
“The article questioned the popularity of jazz compared to that of classical music.”
“The challenger weighed 20 pounds less than the defender did,” or “The challenger’s weight was 20 pounds less than that of the defender.”
There are two ways to read this sentence as it’s written. We could be comparing the Boston office’s contribution to national sales and other branches’ contributions, but we also could be comparing the Boston office’s contribution to national sales and the Boston office’s contribution to those other branches. The former comparison is more logical, so we clarify: “… than any other U.S. branch does.”
A similar problem—you need to repeat the verb after than (“than he paid to the others”) or refer to the verb by placing to after than (“than to the others”).
Now let’s use the Kaplan Method on a Sentence Correction question dealing with comparisons:
Like most other marsupial species and all other kangaroo species, the diet of the swamp wallaby consists of leaves and other sorts of vegetation.
Like most other marsupial species and all other kangaroo species, the diet of the swamp wallaby consists
Like those of most other marsupial species and all other kangaroo species, the diets of the swamp wallaby consists
Just like the diet of most other marsupial species and all other kangaroo species, the diet of the swamp wallaby consists
Similar to the diets of most other marsupial species and all other kangaroo species, the swamp wallabies have a diet which consists
Like most other marsupial species and all other kangaroo species, the swamp wallaby has a diet consisting
This sentence begins with the word “Like,” which signals that you have a comparison to check. Look at the items being compared and make sure they are comparable. In the original sentence, species are compared to a diet. That’s an incorrect comparison.
All the answer choices contain “like” or “similar to,” so they all contain comparisons. As you examine each choice, look to ensure that it makes a proper comparison. Notice how Attention to the Right Detail—to the words that signal a comparison—has helped you to quickly form a strategy for this question.
The incorrect comparison in the original sentence means that you would automatically eliminate (A). Aside from sounding awkward, (B) contains a subject/verb agreement problem—the plural noun “diets” takes the singular verb “consists,” which is incorrect. (C) should be eliminated because in the introductory phrase “diet” should be plural; also, there’s no reason to use “Just like” rather than “Like.” In addition, “Like” is preferable to “Similar to” in (D), which also incorrectly compares diets to wallabies. Only (E) correctly compares the swamp wallaby to other species.
TAKEAWAYS: COMPARISONS
Items being compared must be both grammatically and logically comparable.
Compared items must have parallel form.
The key to handling comparison questions is to make sure that the comparisons are correct and avoid any possibility of ambiguity.
Don’t let intervening phrases or clauses distract you from what is being compared.
Answers and explanations at end of chapter
Recent surveys indicate that, contrary to popular belief, total abstinence from alcohol does not correlate as strongly with good health as with moderate drinking.
as strongly with good health as with moderate drinking
strongly with good health, like moderate drinking does
as strongly with good health as does moderately drinking
as strongly with good health as does moderate drinking
as strongly with good health as moderate drinking
Unlike its fellow Baltic nations, Latvia and Lithuania, the economy of Estonia grew at an astonishing rate in the late 1990s, and at the end of the decade it was placed on the fast track to join the European Union.
its fellow Baltic nations, Latvia and Lithuania, the economy of Estonia grew at an astonishing rate in the late 1990s, and at the end of the decade it was placed
its fellow Baltic nations, Latvia and Lithuania, Estonia grew at an astonishing rate economically in the late 1990s, and at the end of the decade earned itself a place
its fellow Baltic nations, Latvia and Lithuania, Estonia’s economy grew at an astonishing rate in the late 1990s, and at the end of the decade they were placed
Latvia and Lithuania, its fellow Baltic nations, the economy of Estonia grew at an astonishing rate in the late 1990s, and at the end of the decade it was placed
its fellow Baltic nations, Latvia and Lithuania, Estonia experienced economic growth at an astonishing rate in the late 1990s, and at the end of the decade it earned a place
No less significant than international pressures are the constraints that domestic culture and ideology impose on decision making by national political figures.
No less significant than
The things that are just as significant as
Just like the significant
Not lesser than the significance of
What are as significant as
Golden and Labrador Retrievers have enjoyed wide popularity as guide dogs because their dispositions are more suited to companion work than most other breeds.
than
than is true of
than are those of
in comparison to
as compared to
Answers and explanations at end of chapter
Unlike other primates, which are born with fully formed craniums, a newborn human baby’s cranium consists of eight bones that take years to fuse together fully, allowing the brain to grow much larger during those early years.
fully formed craniums, a newborn human baby’s cranium
fully formed craniums, newborn human babies have craniums that
a fully formed cranium, a human baby’s cranium
fully formed craniums, a human is born with a cranium that
a fully formed cranium, the cranium of a newborn human baby
The legal considerations that have forced some universities to revise their affirmative action admissions programs are similar to the revisions that certain large businesses have recently made to their affirmative action hiring practices.
the revisions that certain large businesses have recently made to their affirmative action hiring practices
the affirmative action hiring practices that certain large business recently have revised
those that have recently forced certain large businesses to revise their affirmative action hiring practices
those recent revisions that have been made by certain large businesses to their affirmative action hiring practices
what certain large businesses have done to their affirmative action hiring practices in the recent past.
During the Civil War, nearly three times as many Americans lost their lives from infections that could have been prevented with antiseptic techniques than were killed on the battlefield.
than
than those who
than the number who
as the number who
as
The frequently tested principle of idioms concerns forms of expression that have established themselves in standard English as the “right” way to say things. There’s no grammar rule that applies here; it’s just that these particular expressions are generally agreed upon as correct by English speakers.
If you’re a native English speaker, you probably know most idioms already. You’ll hear them and be able to instinctively tell what’s “right” and what’s not. For example, if someone were to say to you, “I am aware about these problems, because I saw the newscast yesterday,” you’d pick up on the error right away. The correct idiom is “aware of,” not “aware about.” But some idioms are commonly, though incorrectly, used in informal English. For example, your brother says to you, “Here’s a tip—Mom prefers platinum over gold.” Did you spot the error? The correct idiom is “prefers X to Y.” So it’s a good idea to review idioms and to keep a list of those you encounter but don’t know.
If you’re a nonnative English speaker, you’ll likely know fewer idioms. Jot down all the idioms that you come across and make sure you know all the correct forms. Also, look out for the following:
Prepositions (to, from, at, over, etc.) in the underlined portion of the sentence. Their usage is often dictated by idiomatic rules.
Verbs whose idiomatic usage you’ve seen frequently tested. Common examples are prefer, require, and regard.
You will also need to know how to correct problems with usage and style. As tested on the GMAT, usage and style questions reward students for correcting both the syntax and diction of sentences. The GMAT will always prefer active voice and clear, direct style. Look out for the following:
Unneccessary passive voice: “John wrote the letter” is better than “The letter was written by John.”
Unnecessary wording and redundancy. For example, the word “because” is better than the phrase “in view of the fact that.”
Awkward, choppy, or clunky phrasing: use your “ear” to help you identify sentences and phrases that simply don’t read well.
Answers follow the exercises.
Fill in the blank with the correct word or phrase that completes the idiom correctly in the sentence. In some cases, the correct idiom may not require any additional words.
He modestly attributed his business’s success ___________ good luck.
My dictionary defines “idiom” ___________ the usual way in which the words of a particular language are joined together.
Alexander Graham Bell is credited ___________ inventing the telephone.
Some color-blind people cannot distinguish red ___________ green.
Other color-blind people cannot distinguish ___________ yellow ___________ blue.
Although his story seems incredible, I believe it ___________ the truth.
She is regarded ___________ an expert on public health policy.
He is considered ___________ a close friend of the president.
I like to contrast my plaid pants ___________ a lovely paisley jacket.
According to Aristotle, contentment is different ___________ happiness.
The oldest rocks on Earth are estimated ___________ 4.6 billion years old.
Louisiana’s legal system is modeled ___________ the Napoleonic code.
I don’t mean for my comments to be perceived ___________ criticism.
Cigarette ads aimed ___________ children have been banned by the FDA.
The mass extinction of dinosaurs has been linked ___________ a large meteor impact.
Don’t worry ___________ all GMAT idioms; just memorize the ones that your ear doesn’t recognize.
Answers follow the exercises.
Fill in the blank with the correct word or phrase that completes the idiom correctly in the sentence. In some cases, the correct idiom may not require any additional words.
I sold more glasses of lemonade ___________ my neighbor sold.
She sold as many glasses of lemonade ___________ she could.
The bigger they come, ___________ they fall, or so it is said.
According to my diet, I can have either cake or/and ice cream, but not both.
Given my choice, I would have both cake ___________ ice cream.
I must decide between one ___________ the other.
Neither the coach ___________ the players was/were happy with the team’s performance.
I couldn’t decide if/whether he was kidding or not.
Between/Among the three candidates, he has the more/most impressive record.
There are less/fewer students in class today than there were yesterday.
However, the amount/number of students enrolled in this class has increased.
People are forbidden from entering/to enter the park at night.
The ruling prohibits the defendant from discussing/to discuss the case.
Most politicians do not want to be seen associating with/among convicted felons.
We should treat others as/like we would want them to treat us.
I would prefer a salty treat like/such as potato chips over/to a candy bar.
Scores on the GMAT range from 200 ___________ 800.
Most local residents view the monument ___________ an eyesore.
In the United States, there is less opposition ___________ the use of genetically modified foods than in Europe.
Stress can lower one’s resistance ___________ cold and flu viruses.
The rise in inflation has become so significant ___________ constitute a threat to the economic recovery.
The actress’s performance was so poignant ___________ the entire audience was moved to tears.
Just try ___________ do as well as you can on the test.
He modestly attributed his business’s success to good luck.
My dictionary defines “idiom” as the usual way in which the words of a particular language are joined together.
Alexander Graham Bell is credited with inventing the telephone.
Some color-blind people cannot distinguish red from green.
Other color-blind people cannot distinguish between yellow and blue.
Although his story seems incredible, I believe it to be the truth.
She is regarded as an expert on public health policy.
He is considered a close friend of the president. [This blank takes nothing.]
I like to contrast my plaid pants with a lovely paisley jacket.
According to Aristotle, contentment is different from happiness.
The oldest rocks on Earth are estimated to be 4.6 billion years old.
Louisiana’s legal system is modeled after the Napoleonic code.
I don’t mean for my comments to be perceived as criticism.
Cigarette ads aimed at children have been banned by the FDA.
The mass extinction of dinosaurs has been linked to a large meteor impact.
Don’t worry about all GMAT idioms; just memorize the ones that your ear doesn’t recognize.
I sold more glasses of lemonade than my neighbor sold.
She sold as many glasses of lemonade as she could.
The bigger they come, the harder they fall, or so it is said.
According to my diet, I can have either cake or ice cream, but not both.
Given my choice, I would have both cake and ice cream.
I must decide between one and the other.
Neither the coach nor the players were happy with the team’s performance.
I couldn’t decide whether he was kidding or not.
Among the three candidates, he has the most impressive record.
There are fewer students in class today than there were yesterday.
However, the number of students enrolled in this class has increased.
People are forbidden to enter the park at night.
The ruling prohibits the defendant from discussing the case.
Most politicians do not want to be seen associating with convicted felons.
We should treat others as we would want them to treat us.
I would prefer a salty treat such as potato chips to a candy bar.
Scores on the GMAT range from 200 to 800.
Most local residents view the monument as an eyesore.
In the United States, there is less opposition to the use of genetically modified foods than in Europe.
Stress can lower one’s resistance to cold and flu viruses.
The rise in inflation has become so significant as to constitute a threat to the economic recovery.
The actress’s performance was so poignant that the entire audience was moved to tears.
Just try to do as well as you can on the test.
Now let’s use the Kaplan Method on a Sentence Correction question dealing with idioms and usage:
Growth in the industry is at an all-time low, with total employment at less than 68,000 people and fewer companies in the field than there have been during any of the past ten years.
at less than 68,000 people and fewer
at fewer than 68,000 people and fewer
lesser than 68,000 people and fewer
fewer than 68,000 people and less
at less than 68,000 people and there are less
Notice that the underlined portion of the sentence contains the words “less” and “fewer.” These words are often tested on the GMAT because they are often used incorrectly, so be sure to pay attention to them. Doing so will save you valuable time. “Less” can only refer to non-countable items, such as soup or confidence. “Fewer” must refer to countable items, such as chairs or peanuts. In this sentence, since “people” and “companies” can both be counted, “fewer” is appropriate.
Even if that didn’t occur to you as the issue in the sentence, you should note that some of the choices use “less” (or “lesser”) and others use “fewer” and group accordingly into a 3-2 split.
You would eliminate choice (A) because the original sentence uses the non-countable adjective “less” to describe the countable noun “people.” Choices (C) and (E) make the similar errors (and choice (C)’s “lesser than” is thoroughly unidiomatic), so eliminate them as well. Choice (D) uses “less” to describe the countable noun “companies,” so it is incorrect. Only in choice (B) are both countable nouns modified by the countable adjective “fewer,” making it the correct answer.
TAKEAWAYS: IDIOMS AND USAGE
Avoid verbosity and redundancy.
Avoid passive verbs wherever possible.
Avoid clunky, choppy, or awkward-sounding sentences.
Use the correct idioms.
There really are no rules when it comes to idioms, which are simply word combinations that, based on common usage, have established themselves as correct in standard written English. Many GMAT idiom questions require you to use the right prepositions.
Answers and explanations at end of chapter
When a product costs more, be it a bottle of wine or a handbag, the more likely it is to be esteemed by consumers.
When a product costs more, be it
When a product costs more, whether it is
As a product becomes more and more costly, like
The more a product costs, like
The more a product costs, whether it is
Mary Shelley is widely credited for the invention of a new literary genre; many scholars consider her first and only novel, Frankenstein, to be the first science fiction story.
for the invention of a new literary genre; many scholars consider her first and only novel, Frankenstein, to be
for inventing a new literary genre; many scholars consider her first and only novel, Frankenstein, as being
to have invented a new literary genre; many scholars regard her first and only novel, Frankenstein, to be
with the invention of a new literary genre; many scholars regard her first and only novel, Frankenstein, to be
with the invention of a new literary genre; many scholars consider her first and only novel, Frankenstein,
The percentage of people aged 25–44 living alone increased abruptly between 1990 and 1995 and continued to rise more slowly over the next five years.
The percentage of people aged 25–44 living alone increased abruptly between 1990 and 1995 and
There was an abruptly increased percentage of people aged 25–44 who lived alone between 1990 and 1995 and they
The percentage of people aged 25–44 who lived alone increased abruptly between 1990 and 1995 and has
There has been an abrupt increase in the percentage of people aged 25–44 living alone between 1990 and 1995 and it
Between 1990 and 1995, there was an abrupt increase in the percentage of people aged 25–44 who lived alone which
Answers and explanations at end of chapter
The macabre nature of Macbeth, together with the widespread belief that real-life tragedies have accompanied many productions, has made the name of the play so dreaded that not even the least superstitious members of most casts dare utter it.
has made the name of the play so dreaded that not even the least superstitious members of most casts dare
have made the name of the play so dreaded, even the least superstitious members of most casts will not dare to
have made the play’s name sufficiently dreaded, so that even the less superstitious members of most casts will not dare to
have made the name of the play sufficiently dreaded, so that not even the least superstitious members of most casts dare
has made the name of the play is dreaded, so that even the least superstitious members of most casts dare not
The university’s board of trustees, being worried over declining student enrollments and their failing to secure additional funding from the state, has formed a committee to determine what cuts need to be made to staff and programs.
being worried over declining student enrollments and their failing
worrying over declining student enrollments and also the failure
worried about declining student enrollments and the failure
in that they are worried about the decline in student enrollments and the failure
because of its worry concerning the decline of student enrollments and, as well as concerning the failure
Most Sentence Correction errors fall into the categories we’ve just reviewed. Through our research, we’ve seen three other types of errors that show up with enough frequency on the GMAT to merit a word here.
The GMAT likes to test your understanding of the use of the relative pronouns which, where, and when. While the usage of these words may be flexible in informal speech, there are specific rules for how they are used on the GMAT:
On the GMAT, “which” needs to be preceded by a comma and refer to the noun just before the comma, or else its use is wrong (except when it follows a preposition, as in the phrase “in which”).
On the GMAT, “where” must refer to an actual location, or else its use is wrong.
On the GMAT, “when” must refer to an actual time or else its use is wrong.
The subjunctive mood is the form of a verb used to describe a situation that is contrary to fact. You will see the subjunctive mood used in sentences that describe an order or recommendation. The subjunctive mood is also used in sentences that present hypothetical situations. Here are the rules to keep in mind when you see one of these types of sentences:
Orders and recommendations: With verbs such as order, recommend, demand, insist, etc., what follows the verb should be that, a new subject, and the infinitive form of a verb but without the “to.” Example: He demanded that the door be opened.
Hypothetical situations: When contemplating hypothetical or contrary-to-fact situations, use were and would. Example: If I were rich, I would quit my job.
The GMAT uses compound and complex sentences to test your knowledge of clauses and connectors. Recall that a clause is a group of words that contains a subject and a verb. When two clauses appear in a sentence, the clauses should be connected by one, and only one, connector (because, although, as, but, etc.). The connector used should make sense with the rest of the sentence.
Answers follow this exercise.
For each of the following sentences, correct the error related to one of the three listed categories of miscellaneous errors.
After the mediation session, the victim said he was now in a state where he could forgive the offender.
The order from the vice president was that all employees will work through the weekend to finish the project.
If she was more competent, she would be promoted more quickly.
Many citizens agree that austerity measures are necessary, few are happy about the ones that were enacted.
She invited me to join her at the sushi restaurant which just opened next to Carlotta’s Bistro on Main Street.
“Where” can only refer to a place, and “a state (of mind)” is not a place. Use “in which” instead.
The verb in an order should be in the infinitive form, without the “to”, so “to work” simply becomes “work”; there is no need for the “will”: The order from the vice president was that all employees work through the weekend to finish the project.
Use “were” and “would” with hypothetical situations: If she were more competent, she would be promoted more quickly.
The two clauses in this sentence need to be related by a connector: Many citizens agree that austerity measures are necessary, although few are happy about the ones that were enacted.
“Which” is only correct when it appears immediately after a comma or as part of a phrase like “in which.” “That” is the correct word in this case: She invited me to join her at the sushi restaurant that just opened next to Carlotta’s Bistro on Main Street.
Now let’s use the Kaplan Method on a Sentence Correction question dealing with miscellaneous errors:
More and more couples wait before trying to start a family, the average age of first-time parents is increasing.
More and more couples wait before trying to start
As more and more couples wait before trying to start
As more and more couples wait before trying and starting
Although more and more couples wait before trying to start
Being that more and more couples are waiting before trying to start
This sentence contains two complete clauses, each with its own subject and verb. The subject of “wait” is “couples,” and the subject of “is increasing” is “age.” Two complete clauses should be joined with a conjunction, but this sentence has no conjunction and is thus a run-on. The correct answer will provide the sentence with a conjunction. Choice (A) is incorrect.
Choices (B) and (C) introduce “As” at the beginning of the sentence. Choice (D) substitutes “Although,” and choice (E) uses “Being that.”
The GMAT testmakers consider “Being that” to be awkward and wordy, and an answer choice containing it will invariably be wrong. Recognizing this pattern makes choice (E) easy to eliminate. Choice (D) adds the conjunction “Although,” which certainly sounds better than “Being that” but which indicates a contrast and therefore doesn’t make logical sense. “Because” would be a more appropriate connector than “Although.” Choices (B) and (C) both start with “As,” which does make logical sense as a conjunction for this sentence. Choice (C) changes “trying to start” to “trying and starting,” which is an idiomatically incorrect construction. The correct answer is choice (B), which adds an appropriate conjunction and does not create any new errors.
TAKEAWAYS: MISCELLANEOUS ERRORS
Memorize the rules for “which,” “where,” and “when” and apply them with rigor on Test Day.
Memorize the situations in which the subjunctive is used and the rules for how to handle them.
Clauses and connectors must be used correctly on the GMAT. Avoid missing or redundant connectors.
Answers and explanations at end of chapter
The United States’ trade deficit with China rose in 2003 to $123 billion, which is 17 percent more than the previous year and more than 10 times the U.S.-China trade deficit in 1998.
which is 17 percent more than the previous year
which is 17 percent higher than it was the previous year
17 percent higher than the previous year’s figure was
an amount that is 17 percent more than the previous year was
an amount that is 17 percent higher than the previous year’s figure
The two-party political system is one where the electorate gives its votes largely to only two major parties and where one or the other party can usually win a majority in the legislature.
where the electorate gives its votes largely to only two major parties and where
in which the electorate largely gives only its votes to two major parties and where
where the electorate gives largely its votes to only two major parties and in which
in which the electorate gives its votes largely to only two major parties and in which
in which the electorate largely gives only its votes to two major parties and in which
Most financial advisers recommend that stock portfolios should be reviewed at least once a year, if only to make sure that changes do not need to be made.
recommend that stock portfolios should be reviewed
recommend you to review your stock portfolio
recommend that stock portfolios be reviewed
are recommending that stock portfolios are reviewed
have a recommendation to review stock portfolios
The United States would achieve a 10 percent reduction in gasoline consumption, if Congress will raise fuel economy standards to 31.3 miles per gallon for passenger cars and to 24.5 mpg for light trucks.
would achieve a 10 percent reduction in gasoline consumption, if Congress will raise
will achieve a 10 percent reduction in gasoline consumption, if Congress were to raise
will have achieved a 10 percent reduction in gasoline consumption, if Congress will raise
would achieve a 10 percent reduction in gasoline consumption, if Congress were to raise
would achieve a 10 percent reduction in gasoline consumption, if Congress were raising
Because the moon’s distance from Earth varies, so the gravitational pull between the two bodies differ, as do the size and times of oceanic tides.
so the gravitational pull between the two bodies differ
so the gravitational pull between the two bodies differs
the gravitational pull between the two bodies differs
therefore the gravitational pull between the two bodies differ
therefore the gravitational pull between the two bodies differs
Answers and explanations at end of chapter
With a boiling temperature of –195.8 degrees Celsius, nitrogen composes approximately 78 percent of the volume of the atmosphere.
nitrogen composes approximately 78 percent of the volume of the atmosphere
nitrogen is composing the volume of 78 percent of the atmosphere
the atmosphere is approximately 78 percent nitrogen, as measured by volume
nitrogen is composed of 78 percent of the atmosphere’s volume
the atmosphere is composed, in terms of volume, of 78 percent nitrogen
At more than 800 pages in length, the novelist’s life stands revealed in the new biography, which sets the standard for research.
novelist’s life stands revealed in the new biography, which sets the standard for research
new biography is setting standards of research on the novelist’s life
standard for research has been set by a new biography on the novelist
new biography about the novelist sets the standard for research
novelist’s life is revealed in a standard-setting new biography
Because women buy approximately 80 percent of ties sold in the United States, they are often displayed near perfume or women’s clothing departments.
they are often displayed
ties are often being displayed
the displaying of ties is often
ties are often displayed
they often can be found
Although the British were responsible for the early European settlement of both Australia and the United States, Australia is having much closer political and cultural links with Britain than the United States is having.
of both Australia and the United States, Australia is having much closer political and cultural links with Britain than the United States is having
in both Australia and the United States, they are closer in their political and cultural links in Australia than in the United States
of both nations, Australia has much closer political and cultural links to Britain than the United States has
in both nations, Australia is politically and culturally linked to Britain in a much closer fashion than the United States
of both Australia and the United States, they have much closer political and cultural links to it than the United States has
Each of the major setbacks the Germans suffered in June 1944—the fall of Rome, the collapse of Army Group Center on the Eastern Front, and the breaching of the Atlantic Wall at Normandy—were powerful shocks signaling the inability of the German war machine to compete on multiple fronts.
Each of the major setbacks the Germans suffered in June 1944—the fall of Rome, the collapse of Army Group Center on the Eastern Front, and the breaching of the Atlantic Wall at Normandy—were powerful shocks
The fall of Rome, the collapse of Army Group Center on the Eastern Front, and the breaching of the Atlantic Wall at Normandy—each of them major setbacks the Germans suffered in June 1944—were powerful shocks
Powerful shocks—the fall of Rome, the collapse of Army Group Center on the Eastern Front, and the breaching of the Atlantic Wall at Normandy—each a major setback for the Germans in June 1944, was such
The major setbacks suffered by the Germans in June 1944—the fall of Rome, the collapse of Army Group Center on the Eastern Front, and the breaching of the Atlantic Wall at Normandy—each were powerful shocks
Each of the major setbacks the Germans suffered in June 1944—the fall of Rome, the collapse of Army Group Center on the Eastern Front, and the breach of the Atlantic Wall at Normandy—was a powerful shock
Adult pelicans store fish in a deep, expandable pouch below the lower mandible, of which the young feed.
of which the young feed
being for the feeding of the young
from which the young feed
that it feeds the young from
young feed from
Anthony Trollope, one of the most famous English novelists of the nineteenth century, also wrote travel books, some based on his undertaking of journeys while an employee of the postal service.
wrote travel books, some based on his undertaking of journeys
wrote travel books, some based on journeys he undertook
undertook journeys, having based travel books on them
based travel books on journeys undertaken
wrote travel books, some based on journeys which he was to undertake
A federal government survey taken in 1997 showed that during the 12 months preceding the survey, 36 percent of the U.S. population had tried marijuana, cocaine, or other illicit drugs, 71 percent of the population were smoking cigarettes, and 82 percent tried alcoholic beverages.
were smoking cigarettes, and 82 percent
had smoked cigarettes, and 82 percent had
smoked cigarettes, and that 82 percent
smoked cigarettes, and 82 percent had
will be smoking cigarettes, and 82 percent
Like Art Nouveau jewelry designers, the Art Deco movement used art materials suitable for expressing the new stylistic language of the 1920s, one dominated by an interplay of geometric forms.
Art Deco movement used art materials suitable for expressing
Art Deco movement was expressed through the use of materials suitable for the
artists of the Art Deco movement used materials suitable for expressing
Art Deco movement’s materials were used by artists to suitably express
artists of the Art Deco movement also used suitable materials for the expression of
Amtrak, a government-owned corporation, schedules passenger rail service, payments to privately owned firms to run trains, and bears all administrative costs, such as those incurred by the purchase of new equipment and by the sale of tickets.
payments to privately owned firms to run trains, and bears all administrative costs, such as
pays privately owned firms to run trains, and is bearing all administrative costs, such as
pays privately owned firms to run trains, and bears all administrative costs, such as
pays privately owned firms to run trains, and bears all administrative costs, being inclusive of
pays privately owned firms to run trains, and the bearer of all costs of administration, for example
In feudal Korea, cattle were traditionally used as beasts of burden, rather than as a food source, and fish were the primary source of protein in the typical diet.
cattle were traditionally used as beasts of burden, rather than as a food source, and fish were
cattle were being used as beasts of burden, rather than as a food source, and fish were providing
cattle were traditionally used as beasts of burden, rather than a food source, with fish being
the traditional use of cattle was as beasts of burden, rather than for the provision of food, and fish were
cattle were traditionally used for beasts of burden, rather than for a food source, and fish were
The term “support staff” is often used to describe employees that perform lower-level tasks but also are providing essential administrative duties for executive-level managers.
that perform lower-level tasks but also are providing
performing lower-level tasks but also providing
who perform lower-level tasks, but are also providing
that performs lower-level tasks but also provide
who perform lower-level tasks but also provide
National Women’s History Month began as a single week and as a local event when, in 1978, Sonoma County, California, sponsored a women’s history week to promote the teaching of women’s history—a neglected subject in elementary and high school curricula at that time.
began as a single week and as a local event when, in 1978, Sonoma County, California, sponsored
began as a single week local event sponsoring, in Sonoma County, California,
begins with a single week in 1978 sponsoring the local Sonoma County, California of
will have began in 1978 as a single week local event when Sonoma County, California was in sponsorship of
was begun in 1978 when, in Sonoma County, California, there was to be a sponsorship of
People with schizophrenia experience miscarriages at a higher rate of frequency than people without schizophrenia, whose overactive immune systems tend to be indiscriminately rejecting the foreign DNA of the fetus.
People with schizophrenia experience miscarriages at a higher rate of frequency than people without schizophrenia, whose overactive immune systems tend to be indiscriminately rejecting
Miscarriages are more common among people with schizophrenia than among people without schizophrenia, being that their overactive immune systems tend to indiscriminately reject
Miscarriages are more common between people with schizophrenia than they are between people without schizophrenia because the overactive immune systems of people with schizophrenia tend to be indiscriminately rejecting
The miscarriage rate of people with schizophrenia is higher than the rate for people without schizophrenia because the former’s overactive immune systems tend to reject
People with schizophrenia have more common miscarriages than do people without schizophrenia, because their immune systems tend to indiscriminately reject
Working with musician Stevie Wonder as a musical adviser, Ray Kurzweil developed what is now the preferred medium for creating nearly all music for today’s commercial albums, films, and television: a computer-based instrument capable of reproducing musical sounds of a quality once thought to be possible only on grand pianos and other acoustic instruments.
reproducing musical sounds of a quality once thought to be
reproducing musical sounds with a quality as were once thought to be
reproducing in musical sounds a quality that was once thought to be
reproduction of musical sounds whose quality is the same as what were once thought to be
reproduction of musical sounds at a quality once thought of as
Twenty years ago, only 6 percent of students at Dunmore College who intended to major in chemistry were women; today that figure is at least as high as 40 percent.
at least as high as 40 percent
higher than at least 40 percent
higher by 40 percent at the least
40 percent high or more
more higher than 40 percent
E
D
C
A
A
D
C
A
D
D
B
C
C
D
B
D
E
D
A
E
C
C
D
C
B
E
C
B
A
E
B
A
C
D
E
A
C
D
C
E
E
E
A
A
C
E
D
C
D
C
A
D
D
C
E
C
B
B
C
C
A
E
A
D
A
A
1. (E)
The rise in the number of new housing starts in the final two quarters of last year suggest that the sluggish economy should continue its recovery into the first quarter of this year, but weak job growth in the private sector continues to worry some economists.
suggest that the sluggish economy should continue its recovery into the first quarter of this year, but
suggest that the sluggish economy will continue to recover in the first quarter of this year, but that
suggest that the sluggish economy is continuing to recover in the first quarter of this year, but
suggests that the sluggish economy should continue its recovery into the first quarter of this year, but that
suggests that the sluggish economy will continue to recover in the first quarter of this year, but
Step 1: Read the Original Sentence Carefully, Looking for Errors
Whenever the subject or predicate verb of a sentence is underlined, we need to check for subject-verb agreement. In this sentence, the predicate verb, “suggest,” is underlined. The subject of that verb is “rise,” which is singular, so we need a singular verb. However, “suggest” is plural. We’ve discovered a subject-verb agreement error, and the right answer will correct it.
Step 2: Scan and Group the Answer Choices
(A), (B), and (C) retain the error; all use “suggest.” (D) and (E) change the verb to “suggests.”
Step 3: Eliminate Choices Until Only One Remains
Because (A), (B), and (C) all make the same subject-verb agreement mistake, we can eliminate them. The answer will be either (D) or (E). Choice (D) adds the word “that” to the end of the underlined part of the sentence, which changes the meaning. According to the original sentence, the only thing suggested by the rise in housing starts is that the sluggish economy will continue to recover. Adding the word “that” makes it sound as if the rise in housing starts also suggests that weak job growth continues to worry some economists, which is not what the original sentence implied. Because the correct choice will retain the meaning of the original sentence, (D) is incorrect, and (E) is the correct answer.
2. (D)
The green flash, an atmospheric refractive phenomenon whereby the top edge of a setting sun will momentarily turn green, rarely is seen by the naked eye, primarily on account of requiring specific favorable conditions to occur.
on account of requiring
on account of their requiring
because they require
because it requires
because of requiring
Step 1: Read the Original Sentence Carefully, Looking for Errors
The GMAT prefers concise constructions. This underlined piece sounds awkward. Moreover, –ing constructions such as “of requiring” are generally suspect on the GMAT. We’ll seek an answer choice that is less wordy and that removes the phrase “of requiring.”
Step 2: Scan and Group the Answer Choices
(A) and (B) keep “on account of.” (C), (D), and (E) all use “because.”
Step 3: Eliminate Choices Until Only One Remains
“Because” is certainly shorter and more concise than “on account of,” so we’ll eliminate (A) and (B). Choice (C) uses the pronoun “they,” but it refers to the “green flash,” so the pronoun should be singular. Hence, (C) cannot be correct. (D) uses the pronoun “it,” which agrees with its antecedent, the “green flash.” This choice is concise and grammatically correct and is thus the correct answer. (E) is incorrect because it uses the phrase “of requiring,” which we found awkward in the original sentence. Choice (D) is the correct answer.
3. (C)
A new study has found that college students had been more involved in on-campus political activities during the last few years than at any time in the past two decades.
had been more involved in on-campus political activities during the last few years than at any time
had been more involved in on-campus political activities during the last few years than at any other time
have been more involved in on-campus political activities during the last few years than at any other time
have been more involved in on-campus political activities during the last few years than at any time
are more involved in on-campus political activities during the last few years as compared to any other time
Step 1: Read the Original Sentence Carefully, Looking for Errors
This sentence contains an underlined verb, so that’s where we’ll start our investigation. There is no difference between the singular and plural forms of “had been,” so there is no subject-verb agreement error. However, the verb tense does not match the meaning of the sentence. The past perfect tense, indicated by the use of the helping verb “had,” is only appropriate when there are two past events, with one having occurred before the other. In that situation, the past perfect would be used for the earlier of the two events. In the underlined clause of this sentence, though, there is only a single, continuous action—involvement in on-campus political activities. So there should be no use of the past perfect, and the use of the word “had” needs to be corrected.
Step 2: Scan and Group the Answer Choices
(A) and (B) retain the verb “had been involved.” (C) and (D) both use “have been involved,” while (E) uses “are involved.”
Step 3: Eliminate Choices Until Only One Remains
We know that “had been involved” is an incorrect verb tense, so we can eliminate (A) and (B). Choice (E) uses the present tense, but the action being described has been happening “during the last few years,” so we have another inappropriate tense and can eliminate (E) as well. That leaves (C) and (D). Both use the correct tense, the present perfect, to describe an action that occurred in the past over the course of several years. The only difference between them is that (C) adds the word “other” to the phrase “at any time,” while (D) does not. We’ll need to look carefully at the intended meaning of the original sentence to see which is correct. The “last few years” are included in the “past two decades,” so the author of the sentence is really trying to compare the “last few years” to any “other” time during the past two decades. (C) adds this subtle but proper comparative adjective and is therefore the correct answer.
4. (A)
Depending on which scholar you consult, Christopher Columbus, Leif Ericson, or the Chinese eunuch Zheng Ho is credited with being the first explorer from the Eurasian continent to have traveled to the New World by ship.
is credited with being the first explorer from the Eurasian continent to have traveled to the New World by ship
is credited to be the first explorer from the Eurasian continent to have traveled to the New World by ship
is credited to have been the first explorer from the Eurasian continent to have traveled to the New World by ship
are credited with being the first explorers from the Eurasian continent to have traveled to the New World by ship
are credited to be the first explorers from the Eurasian continent to have traveled to the New World by ship
Step 1: Read the Original Sentence Carefully, Looking for Errors
Whenever a sentence contains an underlined verb, we need to make sure that it agrees with its subject; if it does not, there is an error. In this sentence, the predicate verb phrase “is credited” is singular. The subject of the sentence is composed of the names of three different explorers, connected by the conjunction “or,” which means that the verb must agree with that part of the subject that is nearest to the verb. That’s the third explorer, Zheng Ho. Because he’s just one person, we need a singular verb, so “is credited” is correct.
Step 2: Scan and Group the Answer Choices
(A), (B), and (C) keep the verb “is credited.” (D) and (E) both substitute “are credited.”
Step 3: Eliminate Choices Until Only One Remains
Because we know that “is credited” is correct, we can eliminate (D) and (E). Of the remaining choices, (B) changes “with being” to “to be,” and (C) changes “with being” to “to have been.” However, only the original sentence is idiomatically correct: one credits someone “with” doing something. That leaves choice (A) as the correct answer.
5. (A)
Noting that its revenues had fallen due to a recent prolonged slump in CD sales, the music-store chain announced that it would be forced to raise prices at all of its outlets.
its revenues had fallen due to a recent
its revenues have fallen due to a recently
its revenues are falling due to a recently
their revenues are falling due to a recent
their revenues had fallen due to a recent
Step 1: Read the Original Sentence Carefully, Looking for Errors
This sentence contains an underlined verb and a pronoun; we should check both. The verb, “had fallen,” agrees with its plural subject, “revenues.” It is in the past perfect tense, which is appropriate because the revenues fell before the announcement was made. Whenever there are two past events, with one occurring before the other, the chronologically earlier event should be in the past perfect tense. The verb “had fallen,” then, is correct. The singular pronoun “its” refers unambiguously to “the music-store chain,” which is also singular, so the pronoun is correct as well. Chances are that the answer will be (A), but we’ll check the other choices as well to make sure we haven’t overlooked anything.
Step 2: Scan and Group the Answer Choices
(A), (B), and (C) retain the pronoun “its.” (D) and (E) change the pronoun to “their.” There is a second 3-2 split: (A), (D), and (E) keep the adjective “recent,” while (B) and (C) change “recent” to “recently.”
Step 3: Eliminate Choices Until Only One Remains
(D) and (E) can be eliminated because they use a plural pronoun, “their,” to refer to a singular subject, “the music-store chain.” Of the remaining choices, (B) and (C) change the last underlined word from “recent” to “recently.” The original sentence referred to a “prolonged slump” that happened in a “recent” time frame. If the adjective “recent” is changed to the adverb “recently,” this word can no longer describe the noun “slump” (only adjectives can describe nouns). Instead, the adverb “recently” must describe the adjective “prolonged” (i.e., the slump was “recently prolonged”), thereby changing the meaning of the sentence. The correct answer to a Sentence Correction question must match the intended meaning of the original sentence. We can thus throw out (B) and (C). The correct answer remains choice (A).
6. (D)
After all the research that has been conducted in the last 30 years, it is readily apparent that there are, without question, a wealth of good reasons to do aerobic exercise regularly.
there are, without question, a wealth of good reasons to do aerobic exercise regularly
there are, without question, a wealth of good reasons to exercise aerobically regularly
there are, without question, a wealth of good reasons to engage regularly in aerobic exercise
there is, without question, a wealth of good reasons to engage regularly in aerobic exercise
there is, without question, a wealth of good reasons to do regular aerobic exercises
Step 1: Read the Original Sentence Carefully, Looking for Errors
Sometimes the subject of a clause will appear after the verb, as is the case in this sentence. In the underlined subordinate clause, the subject is “a wealth” and the verb is “are”—and both are underlined, so we must check for agreement. (The noun “reasons” cannot be the subject because it is part of the prepositional phrase “of good reasons,” which modifies the subject.) “Wealth” is a singular noun, so we need a singular verb; “are” is plural, so we have a subject-verb agreement error.
Step 2: Scan and Group the Answer Choices
(A), (B), and (C) retain the verb “are.” (D) and (E) both change the verb to “is.”
Step 3: Eliminate Choices Until Only One Remains
We know that “are” is incorrect, so we can eliminate (A), (B), and (C). Choice (E) changes “do aerobic exercise regularly” to “do regular aerobic exercises,” which alters the meaning of the sentence in an illogical way. Here we want the adverb “regularly,” which modifies how often a person should engage in exercise, not the adjective “regular,” which modifies the noun “exercises,” indicating that the exercises themselves should be “regular” or “normal.” Because the correct answer must fix any existing errors without changing the meaning of the original sentence, (E) is incorrect, and (D) is the correct answer.
7. (C)
As state governments become less and less able to support higher education in the coming years, universities have been becoming more and more dependent on alumni networks, corporate sponsorships, and philanthropists.
become less and less able to support higher education in the coming years, universities have been becoming
are becoming less and less able to support higher education in the coming years, universities have become
become less and less able to support higher education in the coming years, universities will become
become less and less able to support higher education in the coming years, universities have become
are becoming less and less able to support higher education in the coming years, universities will become
Step 1: Read the Original Sentence Carefully, Looking for Errors
This sentence contains two underlined verbs. The first, “become,” is plural, and agrees with its subject, “state governments,” which is also plural. The second verb, “have been becoming,” also agrees with its subject, “universities.” However, universities’ increasing dependence on alumni networks will take place “in the coming years,” so we need the future tense, “will become.” We can instantly rule out (A).
Step 2: Scan and Group the Answer Choices
Because we have identified a tense error in the second verb, we’ll use that verb to split the answer choices. (C) and (E) use the future tense, “will become.” (A), (B), and (D) retain some version of a past tense.
Step 3: Eliminate Choices Until Only One Remains
We definitely need the second verb to be in the future tense, so we can throw out (A), (B), and (D). Of the remaining choices, (C) keeps the first verb, “become,” while (E) changes the first verb to “are becoming.” In general, the GMAT testmakers prefer the simplest and most concise constructions. “As state governments become less and less able … in the coming years” makes sense, so there is no need to complicate the verb tense. Choice (C) is the correct answer.
8. (A)
The most common breed of rabbit, Oryctolagus cuniculus, is native to Europe but was accidentally introduced into the Australian continent in the 1850s, causing decades of habitat destruction and crop devastation before its numbers were finally brought under control by bounty hunters.
is native to Europe but was accidentally introduced into the Australian continent in the 1850s, causing
is a native of Europe but was accidentally introduced into the Australian continent in the 1850s, which caused
was a native of Europe but was accidentally introduced into the Australian continent in the 1850s, causing
had been native to Europe but was accidentally introduced into the Australian continent in the 1850s, which caused
had been a native of Europe but was accidentally introduced into the Australian continent in the 1850s, causing
Step 1: Read the Original Sentence Carefully, Looking for Errors
Note the verbs “is” and “was accidentally introduced,” as well as “causing” at the end of the underlined portion. Always pay attention to verbs in the underlined portion of a GMAT Sentence Correction question. Also pay attention to how the underlined portion ends, because that will likely be a source of variation among the answer choices.
Step 2: Scan and Group the Answer Choices
Conducting a quick survey of the beginnings and ends of the answer choices, note the 2-2-1 split among “is,” “had been,” and “was.” Also, (A), (C), and (E) all end with “causing,” while (B) and (D) end with “which caused.”
Step 3: Eliminate Choices Until Only One Remains
First, let’s tackle the verb issue. The rabbit breed is native to Europe; the status of its origins is immutable and does not change because it was introduced into the Australian continent in the 1850s. Therefore, the present tense is correct, making (C), (D), and (E) wrong. And (B) is wrong because the correct idiom to describe a species is “native to” rather than “a native of.” My uncle is a native of Australia; the kangaroo is native to Australia. The sentence is correct as written, making (A) our answer. The author’s use of “causing” is just fine here.
9. (D)
A recent spate of news reports questioning the long-term health benefits of high-fat diets have done little to convince its practitioners that they should follow more traditional weight-loss plans.
have done little to convince its practitioners that they should follow more traditional weight-loss plans
have done little to convince their practitioners to follow more traditional weight-loss plans
has done little to convince its practitioners to follow more traditional weight-loss plans
has done little to convince practitioners of these diets to follow more traditional weight-loss plans
has done little to convince practitioners of these diets they should follow more traditional weight-loss plans
Step 1: Read the Original Sentence Carefully, Looking for Errors
Note the verb in the beginning of the underlined portion. Here the GMAT is testing a common error in a classic way: the test is assessing your ability to determine correct subject-verb agreement by placing the subject as far as possible from the verb to disguise the error. The singular subject of the sentence, “spate,” does not agree with the plural verb “have.”
Step 2: Scan and Group the Answer Choices
Even if you didn’t notice the subject-verb error, seeing the 3-2 split between “has” and “have,” you should get the sense that subject-verb agreement is at issue here.
Step 3: Eliminate Choices Until Only One Remains
The actual subject of the sentence is “spate,” which is singular, so (A) and (B) are out. Reading further, (C) is wrong because “its” is singular while the pronoun’s antecedent, “diets,” is plural. (D) is the correct answer because “convince practitioners … to follow” is idiomatically correct, whereas “convince practitioners … they should follow” is not. If (D) sounded better to you than (E), you should learn to trust your ear on GMAT Sentence Correction questions.
10. (D)
Some economists contend that the way markets react to financial news are based as much on irrational fears and expectations than on sound fiscal analyses.
are based as much on irrational fears and expectations than on
are based as much on their irrational fears and expectations as they are on
is based as much on irrational fears and expectations than on
is based as much on irrational fears and expectations as on
has been based on irrational fears and expectations as much as
Step 1: Read the Original Sentence Carefully, Looking for Errors
Whenever a sentence contains an underlined verb, we must be sure to check whether it agrees with its subject. In this sentence, the verb “are based” is plural, but its subject, “way,” is singular, so we have a subject-verb agreement error. Moreover, the words “as much” are part of the idiomatic phrase “as much … as,” but this sentence instead uses “as much … than.” The correct answer will fix both mistakes without introducing any new errors.
Step 2: Scan and Group the Answer Choices
(A) and (B) retain the plural verb phrase “are based.” (C) and (D) instead use the singular “is based.” (E) uses “has been based.”
Step 3: Eliminate Choices Until Only One Remains
We can eliminate (A) and (B) because we know that the plural verb is incorrect. (E) has a singular verb, but it changes the tense and therefore the meaning of the sentence, so we can rule out (E) as well. (C) and (D) both correctly use the present tense singular “is based.” However, choice (C) does not correct the “as much … than” error. The correct answer is (D), which corrects “as much … than” to “as much … as” and also uses the correct verb form.
11. (B)
The newspaper reported that some homes that were destroyed and severely damaged in the hurricane last year were built too close to the coastline.
some homes that were destroyed and severely damaged in the hurricane last year were
some homes that were destroyed or severely damaged in the hurricane last year had been
some homes the hurricane destroyed and severely damaged last year have been
last year the hurricane destroyed or severely damaged some homes that have been
last year some of the homes that were destroyed or severely damaged in the hurricane have been
Step 1: Read the Original Sentence Carefully, Looking for Errors
This underlined portion contains a lot of verbs. The homes “were destroyed and severely damaged” and “were built” too close to the coast. Did you notice the erroneous “and” in the underlined portion? It’s actually not possible for homes to be destroyed and severely damaged. Homes are either destroyed or severely damaged, not both.
Step 2: Scan and Group the Answer Choices
(A) and (C) use “and,” while the remaining choices use “or.” Note also that the ends of the answer choices present a variety of verb tenses: “were,” “had been,” and “have been.”
Step 3: Eliminate Choices Until Only One Remains
Because “and” is an inappropriate conjunction to use, (A) is out, as is (C), which repeats the error. Since the homes were built first and then destroyed or damaged, the past perfect tense, “had been built,” is appropriate to use here, making (B) the correct answer. (D) and (E) use the present perfect tense, “have been built,” which doesn’t make sense given the sequence of events in the sentence.
12. (C)
The activism of state citizens, who have demanded safer road conditions as well as stiffer penalties for intoxicated drivers, have led to a significant decrease in the number of traffic accidents.
have led to a significant decrease in the number
have led to significant decreases in the amount
has led to a significant decrease in the number
has been significant in the decrease in the number
has significantly decreased the amount
Step 1: Read the Original Sentence Carefully, Looking for Errors
The GMAT will often insert long intervening phrases between commas in order to put distance between a subject and a verb that don’t agree. This sentence is no different. The actual subject of the sentence is “activism,” which is singular, but the underlined portion uses “have led,” a plural verb phrase.
Step 2: Scan and Group the Answer Choices
The answer choices present a choice between “have,” in (A) and (B), and “has,” in (C), (D), and (E), so even if you didn’t notice the error, you could have guessed that subject-verb agreement was an issue here.
Step 3: Eliminate Choices Until Only One Remains
Ignoring the long modifying phrase set off between commas, we see that the subject of this sentence is “activism,” making the singular verb “has” correct, so (A) and (B) are out. There’s also a choice between “number” and “amount” at the ends of the answer choices. Because traffic accidents are countable, the correct word to use is “number,” so (E) is out. Finally, (D) is wordy and a bit awkward (“has been significant in the decrease in”), and it changes the meaning of the sentence. (C) is therefore the correct answer.
13. (C)
The government has imposed sanctions and restricted foreign aid to a renegade nation last month after it violated the terms of a worldwide arms control treaty.
has imposed sanctions and restricted foreign aid to a renegade nation last month after it violated
has imposed sanctions on and restricted foreign aid to a renegade nation last month after it violated
imposed sanctions on and restricted foreign aid to a renegade nation last month after that nation violated
imposed sanctions and restricted foreign aid to a renegade nation last month after violating
had imposed sanctions and restricted foreign aid to a renegade nation last month after it had violated
Step 1: Read the Original Sentence Carefully, Looking for Errors
This sentence contains three errors. First, the underlined pronoun “it” is ambiguous; it is unclear whether it refers to the “government” or to the “renegade nation,” though logically it should refer to the latter. Second, while the predicate verb “has imposed” agrees with its subject, “the government,” the verb tense is incorrect. It makes no sense to say that “the government has imposed sanctions last month.” When we are describing a single past event that occurred at a specific time point, we need the simple past tense: “The government imposed sanctions last month.” Third, the phrase “imposed sanctions” must be followed by the preposition “on” instead of “to” as written. The correct answer must fix the ambiguous pronoun, the faulty verb tense, and the idiomatic phrase “imposed sanctions on” without introducing any new errors.
Step 2: Scan and Group the Answer Choices
(A), (B), and (E) retain the pronoun ambiguity. (C) changes “it” to “that nation,” while (D) removes the pronoun altogether. We have the same 3-2 split for the verb error: (A), (B), and (E) use “has imposed,” while (C) and (D) use “imposed.” There is also a 3-2 split following “imposed sanctions”: (B) and (C) add the word “on,” while (A), (D), and (E) do not.
Step 3: Eliminate Choices Until Only One Remains
We can eliminate (A), (B), and (E) because all three commit the pronoun and verb tense errors we spotted in the original sentence. Of the remaining choices, (D) attempts to fix the ambiguous pronoun by simply leaving it out: “The government imposed sanctions … after violating …” This construction makes it sound as if the same government that imposed the sanctions violated the treaty, which doesn’t make much logical sense. (C) fixes the pronoun ambiguity and also conveys the more logical meaning: “That nation” (i.e., the renegade nation) violated the treaty. Because (C) also fixes the verb tense error by changing “has imposed” to “imposed” and adds the preposition “on” after “imposed sanctions,” it is the correct answer.
14. (D)
Every one of the police reports have been filed so that they can be located by their case number, the date they were created, or the victim’s last name.
have been filed so that they can be located by their case number, the date they were created, or the victim’s last name
have been filed so that they can be located by their case numbers, the dates they were created, or by the victims’ last names
has been filed so that they can be located by their case number, the date they were created, or the victim’s last name
has been filed so that it can be located by its case number, the date it was created, or the victim’s last name
has been filed so that it can be located by its case number, the date it was created, or by the victim’s last name
Step 1: Read the Original Sentence Carefully, Looking for Errors
The subject of this sentence, “Every one,” is singular. However, the verbs and pronouns in the underlined segment are all plural, so there are multiple errors: “Have” should be “has,” “they” and “their” should be “it” and “its,” and “were” should be “was.” Note that the testmaker is trying to trick us into thinking that the subject is “the police reports.” We always need to remember to mentally bracket out prepositional phrases when searching for the subject of a sentence. The “police reports” are, in fact, the object of the preposition “of” and therefore cannot be the subject of the sentence.
Step 2: Scan and Group the Answer Choices
(A), (B), and (C) keep the plural pronouns, while (D) and (E) change “they” and “their” to “it” and “its.” There is a second split among the answer choices with regard to the predicate verb. (A) and (B) retain the plural form “have been,” while (C), (D), and (E) change the predicate verb to the singular “has been.”
Step 3: Eliminate Choices Until Only One Remains
We can throw out (A), (B), and (C) because all three use the incorrect plural pronouns. (D) and (E) both use the singular verb and singular pronouns, so we need to look for some other difference between them. (E) introduces the word “by” into the last phrase: “or by the victim’s last name.” This addition creates a parallelism error. The original sentence contains a list of three different ways to locate a report, and the word “by” is used only once, before the first item in the list. Introducing “by” before the third term makes the second and third terms structurally different from each other so that the items in the list are no longer parallel. Because (D) is the only choice that fixes the errors in the original sentence without introducing any new mistakes, it is the correct answer.
15. (B)
Although swimming can cause cramps in a swimmer’s legs or feet, they can usually be avoided if the swimmer avoids overexertion and performs stretching exercises before entering the pool.
Although swimming can cause cramps in a swimmer’s legs or feet, they
Cramps in a swimmer’s legs or feet caused by swimming
Swimming can cause cramps in a swimmer’s legs or feet, which
The fact that swimming can cause cramps in a swimmer’s legs or feet
Swimming can cause cramps in a swimmer’s legs or feet, although they
Step 1: Read the Original Sentence Carefully, Looking for Errors
It’s not enough to have a good sense of which noun an underlined pronoun refers back to; the sentence has to be 100 percent clear about it. As you were reading, you might have instinctively assumed that “they” refers back to “cramps,” but this isn’t good enough on the GMAT. Because “they” could also refer to “legs” or “feet,” its use here is wrong.
Step 2: Scan and Group the Answer Choices
An initial scan doesn’t yield much in the way of splits, but it’s important to scan for the ways in which the answer choices will deal with the all too common “they” problem.
Step 3: Eliminate Choices Until Only One Remains
(A) is incorrect because the pronoun “they” here is ambiguous; “they” could refer to “cramps,” “legs,” or “feet.” Likewise, (E) is wrong, as is (C), although here the ambiguous pronoun is “which.” What does “which” refer to in choice (C): the legs or feet, the cramps, or the entire preceding clause? (D) is also wrong; a fact cannot be avoided. “The fact that” is another phrase that almost always signals a wrong answer choice on GMAT Sentence Corrections. Finally, (B) is correct because it avoids pronoun reference problems and makes it clear that “cramps” can be avoided.
16. (D)
Some mathematicians argue that to permit a candidate to win an election because they have won a plurality vote is like ranking a student who earned three As and two Fs higher than one who got two As and three Bs.
to permit a candidate to win an election because they have won a plurality vote is like ranking a student
permitting a candidate to win an election because they have won a plurality of votes is like ranking a student
permitting candidates to win elections because they have won a plurality of votes is like to rank a student
permitting candidates to win elections because they have won a plurality of votes is like ranking a student
to permit candidates to win elections because they have won a plurality of votes is like ranking students
Step 1: Read the Original Sentence Carefully, Looking for Errors
This sentence, like many others on the GMAT, contains the underlined pronoun “they.” But here, the plural pronoun “they” refers to “a candidate,” which is singular, so the pronoun agreement is wrong.
Step 2: Scan and Group the Answer Choices
Note that all the answer choices contain “they,” so in order to make the pronoun and antecedent agree, you’ll probably have to change the antecedent to something plural, such as “candidates.”
Step 3: Eliminate Choices Until Only One Remains
You can eliminate (A), as well as (B), because these choices contain the same pronoun agreement error. Examining the remaining choices, note that comparisons require parallel structure, which means that ultimately (C) and (E) must be eliminated in favor of (D): “Permitting … is like ranking …” is correct, whereas comparing a gerund (“permitting” or “ranking”) to an infinitive (“to rank” or “to permit”) is not. Choice (D) is the correct answer.
17. (E)
According to a recently enacted law, any organization that engages in lobbying activities is required either to notify their members about the percentage of dues that are used for lobbying activities or pay a proxy tax.
their members about the percentage of dues that are used for lobbying activities or
their members about the percentage of dues they use for lobbying activities or to
their members about the percentage of dues that are used for lobbying activities or they should
its members about the percentage of dues that are used for lobbying activities or it should
its members about the percentage of dues that is used for lobbying activities or to
Step 1: Read the Original Sentence Carefully, Looking for Errors
This sentence contains several errors. For one thing, the plural pronoun “their” refers to the singular noun “organization.” Pronouns must agree with their antecedents in number, so we need the singular pronoun “its.” There is also a subject-verb agreement issue: “Percentage” is a collective noun referring to a sum of money being spent as a single unit, so it needs a singular verb. However, the verb phrase “are used” is plural. Finally, there is a parallelism error. The word “either” signals a list of two actions, “to notify” and “to pay.” Both actions should be in parallel structure, but the sentence omits the second “to.”
Step 2: Scan and Group the Answer Choices
Let’s use the pronoun at the beginning of the underlined segment to split the choices because it’s the easiest split to see. (A), (B), and (C) retain the plural pronoun “their.” (D) and (E) use the singular pronoun “its.”
Step 3: Eliminate Choices Until Only One Remains
(A), (B), and (C) are out because they use the plural pronoun, which we know to be incorrect. We can also eliminate (D), which does not correct the subject-verb agreement error. That leaves (E), which corrects “are used” to “is used.” Moreover, (E) fixes the parallelism error by inserting the word “to,” so that we now have the correct construction “either to notify … or to pay.” Choice (E) is the correct answer.
18. (D)
The chief executive officer met with the board of directors to consider the possibility of a hostile takeover attempt by a competitor and how they would have to respond to deal with them.
how they would have to respond to deal with them
how to deal with them if action would become necessary
what action would be required for dealing with such an event
what action would be necessary to deal with such an event
the necessity of taking action in order to deal with it
Step 1: Read the Original Sentence Carefully, Looking for Errors
The sentence as written is wrong because “they” and “them” have no clear antecedent nouns. On the GMAT, any hint of pronoun ambiguity indicates an incorrect answer choice.
Step 2: Scan and Group the Answer Choices
Sniff out pronouns in the choices. (A) and (B) use “them,” and (E) uses “it.”
Step 3: Eliminate Choices Until Only One Remains
Because of their ambiguous use of “them,” (A) and (B) are wrong. (C) contains the unidiomatic construction “required for dealing,” and likewise (E) contains the unidiomatic construction “necessity of taking.” Only (D) is idiomatically correct and avoids misused pronouns.
19. (A)
Arguing that the dominance of an economic theory has more to do with the persuasive skills of its expositors than with its accuracy in predicting economic events, Professor McCloskey examines the rhetorical practices of economists.
its expositors than with its
these expositors than its
its expositors compared to its
the theory’s expositors than their
its expositors than with their
Step 1: Read the Original Sentence Carefully, Looking for Errors
There are no obvious problems with the original sentence. The underlined portion uses the pronoun “its” twice, though, so you should ensure that the pronouns agree in number with their antecedents and are not being used ambiguously.
Step 2: Scan and Group the Answer Choices
(A), (B), and (C) use “its” exclusively, while (D) and (E) introduce “their.”
Step 3: Eliminate Choices Until Only One Remains
The sentence is correct as written. Here the pronoun “its” clearly refers to “theory,” so the use is correct. Eliminate (D) and (E), which use “their.” Also, (A) is the only choice that maintains correct parallel structure in the two-part construction “more to do with … than with …” Thus, choice (A) is the correct answer. Eliminate (B) and (C) because they violate the parallel structure.
20. (E)
In an effort to determine whether changes in diet and exercise can reverse coronary heart disease, 1,800 elderly heart patients will participate in a Medicare program costing $12,960,000 over the next three years to reduce lifestyle-related risk factors.
1,800 elderly heart patients will participate in a Medicare program costing $12,960,000 over the next three years to reduce lifestyle-related risk factors
$12,960,000 will be spent by Medicare over the next three years in a program to reduce lifestyle-related risk factors in 1,800 elderly patients
over the next three years $12,960,000 will be spent on 1,800 patients by Medicare on a program to reduce lifestyle-related risk factors
Medicare is to spend $12,960,000 over the next three years on a program for reducing lifestyle-related risk factors in 1,800 elderly heart patients
Medicare will spend $12,960,000 over the next three years on a program to reduce lifestyle-related risk factors in 1,800 elderly heart patients
Step 1: Read the Original Sentence Carefully, Looking for Errors
The intended meaning of this sentence is clear enough: Medicare is making an effort to determine whether lifestyle changes can reverse coronary heart disease. However, the modifying phrase at the beginning of the sentence that describes this effort is followed by the subject of the sentence, “1,800 elderly heart patients.” Grammatically, a modifying phrase modifies whatever is nearest to it in the sentence. So, according to the grammar of this sentence as written, it is the heart patients who are trying to figure out whether lifestyle changes can reverse coronary heart disease—but that’s not the intended meaning. The correct answer would have to either place “Medicare” next to the opening modifying phrase or use a different construction.
Step 2: Scan and Group the Answer Choices
(A), (B), and (C) place something other than “Medicare” next to the opening modifying phrase. (D) and (E) start with “Medicare.”
Step 3: Eliminate Choices Until Only One Remains
(A), (B), and (C) can be eliminated because none of them fixes the misplaced modifier. The only difference between (D) and (E) is the verb phrase: “is to spend” versus “will spend.” Because the program will occur over “the next three years,” we need the future tense, and “will spend” is correct. That makes (E) the correct answer.
21. (C)
Children often pick up on gender norms very early in life, as many parents discover when their toddlers recoil at toys that have only been designed to appeal to toddlers of the opposite sex.
their toddlers recoil at toys that have only been designed to appeal to toddlers of the opposite sex
only their toddlers recoil at toys that have been designed to appeal to toddlers of the opposite sex
their toddlers recoil at toys that have been designed to appeal to toddlers of the opposite sex only
toys that have only been designed to appeal to toddlers of the opposite sex cause their toddlers to recoil
only toys that have been designed to appeal to toddlers of the opposite sex cause their toddlers to recoil
Step 1: Read the Original Sentence Carefully, Looking for Errors
As written, this sentence is illogical. No toy would be designed “only … to appeal to toddlers of the opposite sex.” Toys are designed with lots of functions in mind—entertainment, safety, etc. The modifier “only” is in the wrong place. It should be modifying “toddlers of the opposite sex.” The correct answer will place it properly.
Step 2: Scan and Group the Answer Choices
The choices place the word “only” in a variety of different places. Because this is the only error we’ve spotted, and “only” should go toward the end of the sentence, we expect that the correct answer will still start the same way as the underlined part of the original sentence, so let’s split the choices along these lines. (A) and (C) retain “their toddlers” at the beginning of the underlined segment; choices (B), (D), and (E) do not.
Step 3: Eliminate Choices Until Only One Remains
We know that (A) is incorrect, and chances are that (B), (D), and (E) make erroneous changes, so let’s start by checking choice (C). Here, “only” falls at the very end of the sentence and correctly modifies “toddlers of the opposite sex.” This is the correct answer. For the record: (B) uses “only” to modify “their toddlers,” so that the sentence now excludes any other toddlers. That’s not the intended meaning of the original sentence, so (B) is incorrect. (D) rephrases the underlined segment, but the word “only” still modifies the verb “designed,” so the original error is preserved. (E) uses “only” to modify “toys,” so that the sentence now indicates that nothing besides toys can cause toddlers to recoil. This is again not the intended meaning of the original sentence, so it is also incorrect. Choice (C) is the correct answer.
22. (C)
Based on the candidates’ performances in the televised debates, many pundits predicted that the rival was likely to score an upset defeat over the incumbent senator.
Based on the candidates’ performances in the televised debates
Basing it on the candidates’ performances in the televised debates
Basing their assessments on the candidates’ performances in the televised debates
With the candidates’ performances in the televised debates used as a basis
By the assessments of the candidates’ performances in the televised debates that they made
Step 1: Read the Original Sentence Carefully, Looking for Errors
When a modifying phrase at the beginning of the sentence is underlined, your job is to make sure that the subject of the phrase is what follows the comma. If you read this sentence aloud, you might not have noticed anything because this is a very common error in spoken English. But the subject following the comma is “pundits,” and the modifying phrase “Based on the candidates’ performances in the televised debates” refers not to the pundits but to their predictions. This is a big error.
Step 2: Scan and Group the Answer Choices
There aren’t many obvious splits here, but the opening phrase followed by a comma tells you to eliminate choices based on how they handle the modification error.
Step 3: Eliminate Choices Until Only One Remains
(A) should be eliminated because of the modification error just mentioned. (B) is wrong because the pronoun “it” is misused here, as the reference is unclear and the number should be plural. (D) and (E) are awkward and unidiomatic. (E) is also wrong because it is not clear what “they” refers to. Only (C) is correct, because the subject of the phrase is “pundits,” as it should be, and this choice also avoids pronoun reference problems.
23. (D)
Hoping to avert another situation in which the candidate who won the popular vote could lose the electoral vote, it was proposed by Congress that the Electoral College be abolished.
it was proposed by Congress
there was a proposal by Congress
a proposal was made by Congress
Congress proposed
Congress will have proposed
Step 1: Read the Original Sentence Carefully, Looking for Errors
See that opening phrase followed by a comma? Since that phrase is not underlined, you cannot change it. Instead, you need to make the rest of the sentence align with it. So ask yourself, “Who in the sentence was hoping to avert the situation?” The answer, Congress, should come right after that comma. Anything else is wrong.
Step 2: Scan and Group the Answer Choices
(A), (B), and (C) all begin by talking about the proposal, whereas (D) and (E) begin by talking about Congress.
Step 3: Eliminate Choices Until Only One Remains
Since Congress hoped to avert the situation in the sentence, Congress needs to be placed at the beginning of the underlined portion. This eliminates (A), (B), and (C). Between (D) and (E), the difference lies in verb tense. (E)’s “will have proposed” doesn’t make sense; we would need specific clues in the sentence that require the use of the future perfect tense, and no such clues exist. The simple past, “proposed,” works perfectly well, making (D) the correct answer.
24. (C)
Cheered by better-than-expected opening week box office returns, it was decided by the movie’s producer to give bonuses to the entire cast.
it was decided by the movie’s producer to give bonuses to the entire cast
the entire cast was given bonuses by the movie’s producer
the movie’s producer decided to give the entire cast bonuses
the decision of the movie’s producer was to give the entire cast bonuses
bonuses to the entire cast were given by the movie’s producer
Step 1: Read the Original Sentence Carefully, Looking for Errors
When a sentence starts with a modifying phrase, as this one does, we should be on the alert for misplaced modifiers. The modifying phrase should be placed right next to whatever it logically modifies. In this case, the logic of the sentence dictates that it is the producer who felt “cheered” by the great box office returns. Therefore, “the movie’s producer” should immediately follow the comma. (A) cannot be the correct answer.
Step 2: Scan and Group the Answer Choices
Only (C) places “the movie’s producer” immediately after the comma that follows the modifying phrase. In (B), “the entire cast” follows the comma, while (D) and (E) start out with “the decision” and “bonuses,” respectively.
Step 3: Eliminate Choices Until Only One Remains
We cannot change anything in the part of the sentence that is not underlined, so the opening phrase has to stay where it is. Because we can only rearrange the underlined segment, the only way to fix the modification error is to place “the movie’s producer” right at the beginning of the answer choice. (C) is therefore the correct answer. For the record: (D) and (E) place inanimate things after the comma, which makes no logical sense. A “decision” cannot feel “cheered,” nor can “bonuses.” As for (B), while it may be true that the “entire cast” felt happy about the box office returns, the logical cause-and-effect relationship indicated by the sentence needs to be reflected in its grammar. The producer feels “cheered” and therefore awards the cast bonuses. No matter how the cast feels about the box office returns, the sentence’s initial phrase logically refers to “the movie’s producer.” Choice (C) is correct.
25. (B)
States now have an incentive to lower the blood alcohol level that constitutes drunk driving by a federal law that withholds highway funds from those states that don’t enforce the applicable standard.
that constitutes drunk driving by a federal law that withholds
that constitutes drunk driving, because a federal law withholds
that constitutes drunk driving, because a federal law withheld
which constitutes drunk driving by a federal law that withholds
which constitutes drunk driving, because a federal law withholds
Step 1: Read the Original Sentence Carefully, Looking for Errors
The sentence as written is wrong, because the phrase “drunk driving by a federal law” makes it sound as if the federal law was driving drunk, when the federal law is actually providing an incentive for states to lower their drunk-driving blood alcohol threshold.
Step 2: Scan and Group the Answer Choices
Note that (A), (B), and (C) begin with “that,” while (D) and (E) begin with “which.”
Step 3: Eliminate Choices Until Only One Remains
The original “by a federal law” error disqualifies (A) and (D). On the GMAT, “that” is a restrictive pronoun to be used when the information following “that” is necessary to the sentence. If someone says, “read the book that is on the shelf,” you are presumably in a room that contains multiple books in various locations, so “on the shelf” is necessary information if you want to read the right book. However, if someone says “read the book, which is on the shelf,” you are in a room where there is only one book and that book just so happens to be on the shelf. In this case, “on the shelf” is not necessary information. Back to our sentence. Since the sentence is specifically interested in the blood alcohol level that indicates drunk driving, and not just any old blood alcohol level, “that” is correct because the information is necessary. This eliminates (D) and (E). Finally, (C) can be eliminated because “withheld” is the wrong tense given that the rest of the sentence is in the present tense. Thus, (B) is the correct answer.
26. (E)
Using a seismic survey, hydrocarbons can be located even though they are buried far beneath Earth’s surface.
Using a seismic survey, hydrocarbons can be located even though they are buried far beneath Earth’s surface
Hydrocarbons can be located even though they are buried far beneath Earth’s surface, using a seismic survey
Locating hydrocarbons that are buried far beneath Earth’s surface, a geophysicist can use a seismic survey
Buried far beneath Earth’s surface, hydrocarbons can be detected using a seismic survey
Using a seismic survey, a geophysicist can locate hydrocarbons even though they are buried far beneath Earth’s surface
Step 1: Read the Original Sentence Carefully, Looking for Errors
The original sentence is wrong because it begins with the modifying phrase “using a seismic survey,” so what follows the phrase should be its subject, which is definitely not “hydrocarbons” (it wouldn’t make any sense for hydrocarbons to use a seismic survey).
Step 2: Scan and Group the Answer Choices
These choices don’t scan easily, but since we know that we are dealing with an error with the modifying phrase “using a seismic survey,” we can group choices based on what subject that phrase modifies. (C) and (E) both have the subject “a geophysicist,” which makes sense. (B) and (D) are both vague at best about the subject. In any case, it’s not someone who’d use a seismic survey.
Step 3: Eliminate Choices Until Only One Remains
The modification error disqualifies (A). Choice (B) likewise contains the same misplaced modifying phrase, although here it can be found at the end of the sentence rather than the beginning. (C) sounds awkward and changes the meaning of the sentence, because the seismic survey is precisely what the geophysicist uses to locate the hydrocarbons in the first place. (D) uses a passive verb and never explains who is using the seismic survey. Only (E) correctly places the phrase “Using a seismic survey” next to the subject of the phrase, “a geophysicist.” Thus, choice (E) is the correct answer.
27. (C)
The creation of an independent treasury, establishing lower tariffs, and purchasing the Oregon Territory, all credited to the presidency of James Knox Polk, are among the significant accomplishments that persuade historians to rank this former governor of Tennessee as an above-average president.
The creation of an independent treasury, establishing lower tariffs, and purchasing
The creation of an independent treasury, establishing lower tariffs, as well as purchasing
The creation of an independent treasury, the establishment of lower tariffs, and the purchase of
Creating an independent treasury, the establishment of lower tariffs, and purchasing
Creating an independent treasury, the establishing of lower tariffs, and the purchasing of
Step 1: Read the Original Sentence Carefully, Looking for Errors
The underlined segment of this sentence contains a list of three accomplishments of Polk’s presidency. Items in a list must have parallel form, but these do not: the first term in the list is a noun, “creation,” but the second and third terms are –ing verb forms. The correct answer will either change “The creation of” to “Creating,” or it will change “establishing” and “purchasing” to “the establishment of” and “the purchase of,” respectively.
Step 2: Scan and Group the Answer Choices
(A), (B), and (C) keep the noun “creation” at the beginning of the sentence, while (D) and (E) change “creation” to “creating.”
Step 3: Eliminate Choices Until Only One Remains
(A) is definitely incorrect due to the parallelism issue. We’ll need to check each of the other choices for proper parallel structure. (B) and (C) begin with the word “creation,” so the remaining two items on the list should both be nouns. (B) still has the two gerund forms, “establishing” and “purchasing,” so it does not have the correct parallel structure and cannot be correct. Choice (C), though, gets it right; the three items here are “creation,” “establishment,” and “purchase.” This is the correct answer. A quick look at (D) and (E) reveals that neither has parallel structure. In (D), the middle term is different from the first and third. Choice (E) uses three gerund forms, but there is still a parallelism error. The second and third terms are in the same form, “the establishing of” and “the purchasing of,” but the first term lacks the words “the” and “of.” Choice (C) is correct.
28. (B)
The rapid-rail lines that were recently introduced in southern California have had positive effects: not only have they lowered pollution levels, they have helped in decongesting the region’s freeway system, which had been operating far above its capacity.
not only have they lowered pollution levels, they have helped in decongesting
the lines have not only lowered pollution levels, but also helped to decongest
not only have pollution levels been lowered, but they have helped to decongest
lines have lowered pollution levels, but also helped in decongesting
not only have they lowered pollution levels, but also have they helped decongest
Step 1: Read the Original Sentence Carefully, Looking for Errors
The underlined part of this sentence contains the phrase “not only.” That means that we expect to see the words “but also” elsewhere in the sentence, but those words are absent. Additionally, the sentence includes the phrase “helped in decongesting” where “helped to decongest” would be more idiomatic. We can therefore eliminate (A). The correct answer will fix these problems without introducing any new errors.
Step 2: Scan and Group the Answer Choices
(B), (D), and (E) add the “but also” construction; (C) does not, though it adds the word “but.”
Step 3: Eliminate Choices Until Only One Remains
(C) adds “but,” but it leaves out “also.” (E) has both halves of the proper construction, but the verb phrase at the end, “have they helped decongest,” sounds like a question and is awkward at best. (D) adds “but also” but removes the “not only,” so it only has half the construction. Moreover, (D) omits the article “the” at the beginning, such that the second half of the sentence now seems to refer to “lines” generally, rather than to the new Californian rapid rail lines specifically. That leaves (B), which provides both parts of the “not only … but also” construction. Additionally, (B) uses the idiomatically correct form “helped to decongest.” Choice (B) is the correct answer.
29. (A)
A national study evaluating the need for gun control is currently attempting to document where criminals purchase guns, how gun owners are trained in the use of firearms, and what character traits are common to persons who are liable to turn to violence to resolve problems.
what character traits are common to persons who are liable to turn
what are the character traits common to persons who are liable to turning
the common character traits of persons who are liable to turn
the traits of character common to persons who are liable to turn
the traits of character that are common to persons who are liable to turn
Step 1: Read the Original Sentence Carefully, Looking for Errors
Because this sentence has a list, you have to check for parallel structure. The parallel list is “where criminals purchase … how gun owners are trained … and what character traits are common …”
Step 2: Scan and Group the Answer Choices
(A) and (B) begin with “what,” which is parallel to “where” and “how,” whereas the remaining choices begin with elements that are not parallel.
Step 3: Eliminate Choices Until Only One Remains
(C), (D), and (E) are incorrect because they do not begin the third element in the sequence with “what.” Finally, (B) is wrong because it replaces the correct form “to turn” with the incorrect “to turning.” The sentence here is correct as written, so select (A).
30. (E)
Hearing Kenneth Branagh deliver the “St. Crispin’s Day” speech in Henry V is to be mesmerized by a great performer, with seemingly boundless emotional energy.
Hearing Kenneth Branagh deliver the “St. Crispin’s Day” speech in Henry V is to be mesmerized by a great performer, with seemingly
Hearing Kenneth Branagh deliver the “St. Crispin’s Day” speech in Henry V is to be mesmerized by a great performer, one with seeming
Hearing Kenneth Branagh deliver the “St. Crispin’s Day” speech in Henry V is being mesmerized by a great performer, one with seeming
To hear Kenneth Branagh deliver the “St. Crispin’s Day” speech in Henry V is being mesmerized by a great performer, with seemingly
To hear Kenneth Branagh deliver the “St. Crispin’s Day” speech in Henry V is to be mesmerized by a great performer, one with seemingly
Step 1: Read the Original Sentence Carefully, Looking for Errors
Elements in a comparison or analogy, like items in a list, must exhibit parallel structure. Here, the original sentence is incorrect because the analogy states that “Hearing … is to be mesmerized …” The two ideas compared should both be either gerunds ending in –ing or infinitives. Because the GMAT is not particularly fond of gerunds, expect the right answer to change “hearing” to “to hear.”
Step 2: Scan and Group the Answer Choices
The first three answer choices all begin with “hearing,” while the last two begin with “to hear.”
Step 3: Eliminate Choices Until Only One Remains
Because the elements in the comparison in the original sentence are not parallel, eliminate (A). Choice (B) should be eliminated because it contains the same parallel structure error. Choice (D) also contains a parallel structure error: “To hear … is being mesmerized …” Choice (C) may seem a bit more parallel, but it still sounds awkward: “Hearing is being mesmerized …” (You should usually try to avoid answer choices that contain “being” on GMAT Sentence Corrections.) Moreover, (C) contains an adverb/adjective error: the adjective form “seeming” is incorrect because it modifies the adjective “boundless,” and thus the correct form is the adverb “seemingly.” Finally, (E) is correct because it exhibits parallel structure and uses the correct adverbial form, “seemingly.”
31. (B)
By the time they completed their journey, the young explorers had overcome their fears, sharpened their survival skills, and had developed a healthy respect for nature’s potential destructiveness.
skills, and had developed a healthy respect
skills, and developed a healthy respect
skills and a healthy respect developed
skills, developing a healthy respect
skills, all the while developing a healthy respect
Step 1: Read the Original Sentence Carefully, Looking for Errors
This sentence contains an underlined verb, which means that we must check for subject-verb agreement and correct tense. The verb phrase “had developed” agrees with its subject, “the young explorers.” Because the young explorers’ actions precede the completion of their journey, the use of the past perfect tense is correct. However, the phrase “had developed a healthy respect” is part of a list of three things the young explorers did during their journey, and we must therefore also check for parallel structure. Here there is an error, as the second and third items on the list do not have the same structure. The second verb on the list, “sharpened,” does not repeat the helping verb, “had,” that is used at the beginning of the list to indicate the past perfect tense. But the third verb, “had developed,” does repeat the word “had.” We can rule out (A).
Step 2: Scan and Group the Answer Choices
Of the remaining choices, (B) simply removes “had” but leaves the rest of the underlined segment alone, while (D) and (E) change the verb to “developing.” Like (B), choice (C) uses “developed,” but changes the word order.
Step 3: Eliminate Choices Until Only One Remains
Because we cannot change anything in the sentence except the underlined part, we cannot correct the parallelism error by changing “sharpened” to “had sharpened.” Our only recourse is to change “had developed” to “developed.” (B) does just that without changing anything else, and this will likely turn out to be the correct answer. A quick look at (C) reveals that in changing the word order, it destroys the parallelism of the list of three actions completely. As for (D) and (E), changing “developed” to “developing” changes the tense and again destroys the list’s parallel structure. Choice (B) remains the correct answer.
32. (A)
However much Americans may agree that the financing of elections with special interest money undermines democracy and that campaign finance reform would produce better government, it has been very difficult to push such measures through a Congress that has been elected using the old financing system.
However much Americans may agree that
Despite agreement among Americans to the fact
Although Americans agree
Even though Americans may agree
There is agreement among Americans that
Step 1: Read the Original Sentence Carefully, Looking for Errors
The sentence as written may sound a little unusual, so look for the grammatical issues present before turning to style. The sentence sets up a two-part parallel structure: “that the financing of elections with special interest money undermines … and that campaign finance reform would produce …”
Step 2: Scan and Group the Answer Choices
Only (A) and (E) contain the required first “that.” If you don’t read (B) into the rest of the sentence, your brain might fill in the word “that,” but it actually isn’t present in the answer choice.
Step 3: Eliminate Choices Until Only One Remains
Because only (A) and (E) contain the “that” necessary for the parallel structure, eliminate (B), (C), and (D). Choice (E) is wrong because there is no connector between the two clauses in the sentence, save for a measly comma. (Also note that “There is” sentences are usually wrong on the GMAT.) So even though the construction in (A) is not an obviously correct stylistic choice, there’s no better alternative in the other answer choices. This shows us that we don’t have to be in love with the correct answer; it just has to be grammatically and stylistically correct and more concise than any other answer choice. Thus, (A) is the correct answer.
33. (C)
South Pacific Vacations has a package tour to Sydney, Australia, for $999 per person, including airfare from Los Angeles, spending five nights at the Mega Hotel, and to take a harbor cruise, and round-trip airfare from New York is an additional $400.
spending five nights at the Mega Hotel, and to take a harbor cruise, and round-trip airfare from New York is
spending five nights at the Mega Hotel, and taking a harbor cruise; and in addition round-trip airfare from New York is
five nights at the Mega Hotel, and a harbor cruise; round-trip airfare from New York is
with five nights at the Mega Hotel, and a harbor cruise, with round-trip airfare from New York being
to spend five nights at the Mega Hotel, and to take a harbor cruise; round-trip airfare from New York is
Step 1: Read the Original Sentence Carefully, Looking for Errors
Note that the sentence makes a list of items included in the package tour, so that list should be in parallel form. But each of the three items is in a different form: “Airfare,” “spending five nights,” and “to take a harbor cruise” are about as far from parallel as you can get. Expect the right answer to rectify this by making the other items parallel to “airfare,” since that’s the one item we can’t change.
Step 2: Scan and Group the Answer Choices
(A) and (B) don’t change the form of the second item in the list (“spending five nights”), while (C), (D), and (E) change it in various ways.
Step 3: Eliminate Choices Until Only One Remains
Since the noun “airfare,” the first item in the list, is not underlined, the other items in the list must be changed to agree with this form. The only choice that turns the second item, “spending five nights,” into a simple noun is (C). Each of the other choices fails to exhibit truly parallel structure and should be eliminated on that basis. As a side note, (C) also changes the conjunction “and” before “round-trip airfare” to a semicolon, which is preferable because this part of the sentence starts a new thought.
34. (D)
Recent surveys indicate that, contrary to popular belief, total abstinence from alcohol does not correlate as strongly with good health as with moderate drinking.
as strongly with good health as with moderate drinking
strongly with good health, like moderate drinking does
as strongly with good health as does moderately drinking
as strongly with good health as does moderate drinking
as strongly with good health as moderate drinking
Step 1: Read the Original Sentence Carefully, Looking for Errors
This sentence contains an illogical comparison. The sentence is trying to say that abstinence does not correlate as strongly with good health as moderate drinking does. In other words, it is trying to compare abstinence to moderate drinking in terms of health impact. However, the word “with” before “moderate drinking” creates the rather bizarre comparison between good health and moderate drinking; what the sentence actually says is that abstinence correlates better with moderate drinking than it does with good health, which makes no logical sense. We rule out (A) and proceed to split the remaining choices.
Step 2: Scan and Group the Answer Choices
(C), (D), and (E) retain the comparison, “as strongly,” while (B) uses the comparison word “like” instead of “as.”
Step 3: Eliminate Choices Until Only One Remains
We can eliminate (B) because it uses the wrong word, “like,” to make a comparison between clauses (subject + verb), whereas “like” should be used only to compare nouns. (B) also alters the meaning of the sentence: rather than comparing how strongly total abstinence and moderate drinking correlate with good health, (B) states that only one of the two behaviors correlates strongly with good health. (C) corrects the comparison, but “drinking” is a gerund—a verb being used as a noun—and should therefore be modified by an adjective, not the adverb “moderately.” While (E) leaves out the word “with,” it still commits the same illogical comparison as does the original sentence. Choice (D), like (C), fixes the comparison, this time without introducing any new mistakes. (D) is the correct answer.
35. (E)
Unlike its fellow Baltic nations, Latvia and Lithuania, the economy of Estonia grew at an astonishing rate in the late 1990s, and at the end of the decade it was placed on the fast track to join the European Union.
its fellow Baltic nations, Latvia and Lithuania, the economy of Estonia grew at an astonishing rate in the late 1990s, and at the end of the decade it was placed
its fellow Baltic nations, Latvia and Lithuania, Estonia grew at an astonishing rate economically in the late 1990s, and at the end of the decade earned itself a place
its fellow Baltic nations, Latvia and Lithuania, Estonia’s economy grew at an astonishing rate in the late 1990s, and at the end of the decade they were placed
Latvia and Lithuania, its fellow Baltic nations, the economy of Estonia grew at an astonishing rate in the late 1990s, and at the end of the decade it was placed
its fellow Baltic nations, Latvia and Lithuania, Estonia experienced economic growth at an astonishing rate in the late 1990s, and at the end of the decade it earned a place
Step 1: Read the Original Sentence Carefully, Looking for Errors
“Unlike” signals that this sentence will contain a comparison, so we’ll need to check whether that comparison is logical. In this sentence, it is not: Latvia and Lithuania are being compared to Estonia’s economy. Countries can only be compared to other countries, not to economies. The correct answer will fix this illogical comparison.
Step 2: Scan and Group the Answer Choices
(A) and (D) compare Latvia and Lithuania to “the economy of Estonia.” (C) compares Latvia and Lithuania to “Estonia’s economy.” (B) and (E) compare Latvia and Lithuania to “Estonia.”
Step 3: Eliminate Choices Until Only One Remains
(A), (C), and (D) all commit the error of comparing a country to a country’s economy, so all are incorrect. (B) correctly compares Latvia and Lithuania to Estonia, but it unnecessarily employs a reflexive pronoun, “itself,” and its use of the word “economically” sounds rather awkward. (E) also correctly compares Latvia and Lithuania to Estonia, this time without resorting to unnecessary or awkward constructions. Choice (E) is the correct answer.
36. (A)
No less significant than international pressures are the constraints that domestic culture and ideology impose on decision making by national political figures.
No less significant than
The things that are just as significant as
Just like the significant
Not lesser than the significance of
What are as significant as
Step 1: Read the Original Sentence Carefully, Looking for Errors
The original sentence contains a slightly unusual construction but is nonetheless correct. Because a comparison is made in this sentence, you should see whether the comparison is valid.
Step 2: Scan and Group the Answer Choices
There are no splits here, but be on the lookout for answer choices that alter the comparison in any way.
Step 3: Eliminate Choices Until Only One Remains
The original sentence appears to make a valid comparison between “international pressures” and “constraints,” so (A) is correct. Eliminate (B) and (E) for being unnecessarily wordy. Eliminate (C) because it changes the meaning of the sentence slightly by leaving out the idea that the constraints are equally significant. Finally, eliminate (D) because it compares “significance” with “constraints.”
37. (C)
Golden and Labrador Retrievers have enjoyed wide popularity as guide dogs because their dispositions are more suited to companion work than most other breeds.
than
than is true of
than are those of
in comparison to
as compared to
Step 1: Read the Original Sentence Carefully, Looking for Errors
Even though your ear may not catch the error in this sentence the first time you read it, the comparison in the sentence needs to compare two logically comparable things in order to be correct. Here, “dispositions” are being compared to “breeds,” which is an illogical comparison.
Step 2: Scan and Group the Answer Choices
(A), (B), and (C) begin with “than,” while (D) and (E) use lengthier constructions. Note that the only way one of these can be correct is if one of them corrects the error, and none of the shorter answer choices does.
Step 3: Eliminate Choices Until Only One Remains
Because the sentence needs to compare the dispositions of Golden Retrievers and Labradors with the dispositions of other breeds, you can eliminate (A), (B), (D), and (E) as none of them makes the proper comparison. Only the correct answer, (C), offers a logical comparison.
38. (D)
Unlike other primates, which are born with fully formed craniums, a newborn human baby’s cranium consists of eight bones that take years to fuse together fully, allowing the brain to grow much larger during those early years.
fully formed craniums, a newborn human baby’s cranium
fully formed craniums, newborn human babies have craniums that
a fully formed craniums, a human baby’s cranium
fully formed craniums, a human is born with a cranium that
a fully formed cranium, the cranium of a newborn human baby
Step 1: Read the Original Sentence Carefully, Looking for Errors
The word “Unlike” at the beginning of the sentence signals a comparison, which we must check for logic. To do this, we can leave out the distracting phrase between the commas: “Unlike other primates, a newborn human baby’s cranium …” Whenever we see a comparison on the GMAT, we need to make certain that the sentence compares classes of things that yield meaningful information when placed side by side. In this sentence, it would be logical to compare other primates to humans or other primates’ craniums to humans’ craniums. But as written, the sentence compares other primates to a human baby’s cranium, which makes no logical sense. We have an error and can rule out (A).
Step 2: Scan and Group the Answer Choices
(C) and (E) retain the comparison between other primates and a human cranium. (B) compares “other primates” to “newborn human babies,” and (D) compares “other primates” to “a human.”
Step 3: Eliminate Choices Until Only One Remains
We already know that (A) is incorrect. We can eliminate (C) and (E) because they commit the same illogical comparison as the original sentence. (B) logically compares “other primates” to “newborn human babies.” However, reading (B) back into the original sentence reveals a new error: “newborn babies have craniums that consists …” The verb “consists” is singular, so “craniums” should be singular as well. A choice that introduces a new error is incorrect, so we can rule out (B). Finally, (D) makes a logical comparison between “other primates” and “a human,” and this time, “cranium” is singular and agrees with “consists.” Because (D) fixes the error in the original sentence without introducing any new mistakes, it is the correct answer.
39. (C)
The legal considerations that have forced some universities to revise their affirmative action admissions programs are similar to the revisions that certain large businesses have recently made to their affirmative action hiring practices.
the revisions that certain large businesses have recently made to their affirmative action hiring practices
the affirmative action hiring practices that certain large business recently have revised
those that have recently forced certain large businesses to revise their affirmative action hiring practices
those recent revisions that have been made by certain large businesses to their affirmative action hiring practices
what certain large businesses have done to their affirmative action hiring practices in the recent past
Step 1: Read the Original Sentence Carefully, Looking for Errors
“Are similar to” in the original sentence indicates a comparison, so you have to make sure that similar elements are being compared. Whenever you recognize a comparison, cut through the filler until you’ve assessed whether the elements being compared are logically and grammatically similar. This sentence begins with “legal considerations … are similar to … ,” so what follows should also be “legal considerations.”
Step 2: Scan and Group the Answer Choices
(A) begins with “revisions.” (B) begins with “… hiring practices.” (C) and (D) begin with “those,” a word that could refer to “legal considerations.” (E) begins with “what certain large businesses have done.”
Step 3: Eliminate Choices Until Only One Remains
“Revisions” and “practices” aren’t comparable to “legal considerations,” so eliminate (A) and (B). Choice (D) begins with “those,” but if you read further, you’ll notice that (D) uses “those recent revisions,” which are not comparable to “legal considerations,” so eliminate (D). Finally, (E) is unclear and doesn’t provide a specific noun to compare to “legal considerations.” Only (C) make logical sense and corrects the comparison error.
40. (E)
During the Civil War, nearly three times as many Americans lost their lives from infections that could have been prevented with antiseptic techniques than were killed on the battlefield.
than
than those who
than the number who
as the number who
as
Step 1: Read the Original Sentence Carefully, Looking for Errors
The underlined portion here is only one word: “than.” Note that in the beginning of the sentence, the phrase “nearly three times as many” indicates a comparison. Comparisons require parallel construction. If you remove the intervening words “Americans … antiseptic techniques,” the sentence would read, “nearly three times as many than.” That’s an incorrect construction.
Step 2: Scan and Group the Answer Choices
One choice here is between “than” in (A), (B), and (C) and “as” in (D) and (E), so you would have to decide which one is correct even if you didn’t spot the error while completing Step 1 of the Kaplan Method.
Step 3: Eliminate Choices Until Only One Remains
The correct word is “as,” to complete the construction “as many … as …” Thus (A), (B), and (C) are all out. But the comparison also requires parallel structure to “Americans lost their lives …” “The number” is not parallel to “Americans.” So (D) needs to be eliminated in favor of (E). Choice (E) is the correct answer.
41. (E)
When a product costs more, be it a bottle of wine or a handbag, the more likely it is to be esteemed by consumers.
When a product costs more, be it
When a product costs more, whether it is
As a product becomes more and more costly, like
The more a product costs, like
The more a product costs, whether it is
Step 1: Read the Original Sentence Carefully, Looking for Errors
The underlined segment of this sentence gives us the first half of a comparison: “A product costs more” than something. Unfortunately, the sentence fails to state what the product is being compared to. It costs more than … what? Half a comparison is definitely an error that the correct answer will have to fix. We get a clue as to the proper construction we need from the second half of the sentence. The construction “the more likely” is part of a two-part construction, similar to “not only … but also.” The correct construction is “the more … the more.” This sentence should read, “The more a product costs, … the more likely it is to be esteemed by consumers.”
Step 2: Scan and Group the Answer Choices
(A) and (B) retain the partial comparison. (C) gets rid of the comparison with the phrase, “more and more costly.” (D) and (E) use the construction we’re looking for: “The more a product costs …”
Step 3: Eliminate Choices Until Only One Remains
(A) and (B) are incorrect because both contain the partial comparison that we flagged as an error. We can eliminate (C) because while it attempts a statement of proportionality, it does not use the correct grammatical construction; the sentence needs to start with the phrase “The more.” Both remaining choices use “The more,” so one of them will be the correct answer. (D) uses “like” in place of the original “be it.” The original meaning of the sentence is that it doesn’t matter whether the product in question is a bottle of wine or a handbag (or something else). The use of the word “like” in (D) changes the meaning of the sentence. On the GMAT, “like” is used to mean “similar to,” so this answer choice causes the sentence to refer only to items that are similar to wine and handbags. An answer choice that changes the meaning of the sentence cannot be correct, so the answer must be (E). Indeed, (E) substitutes “whether it is” for “be it,” so that the idea that the identity of the product makes no difference is preserved. Choice (E) is the correct answer.
42. (E)
Mary Shelley is widely credited for the invention of a new literary genre; many scholars consider her first and only novel, Frankenstein, to be the first science fiction story.
for the invention of a new literary genre; many scholars consider her first and only novel, Frankenstein, to be
for inventing a new literary genre; many scholars consider her first and only novel, Frankenstein, as being
to have invented a new literary genre; many scholars regard her first and only novel, Frankenstein, to be
with the invention of a new literary genre; many scholars regard her first and only novel, Frankenstein, to be
with the invention of a new literary genre; many scholars consider her first and only novel, Frankenstein,
Step 1: Read the Original Sentence Carefully, Looking for Errors
The original sentence contains an idiom error—two, in fact. The sentence says Shelley is “credited for” inventing a genre and that her book is “consider(ed) … to be” the first science fiction book.
Step 2: Scan and Group the Answer Choices
The choices have a 2-2-1 split: (A) and (B) begin with “for,” (C) begins with “to have,” and (D) and (E) begin with “with.”
Step 3: Eliminate Choices Until Only One Remains
The correct idiomatic expression is “credited with,” so (A), (B), and (C) are all wrong. Also, you should note that, although many grammarians have no problem using “consider” with “to be,” on the GMAT the correct answer choice will always use “consider” without the “to be.” (D) is wrong because the correct idiomatic expression is “regard as,” not “regard to be.” Only (E) is idiomatically correct.
43. (A)
The percentage of people aged 25–44 living alone increased abruptly between 1990 and 1995 and continued to rise more slowly over the next five years.
The percentage of people aged 25–44 living alone increased abruptly between 1990 and 1995 and
There was an abruptly increased percentage of people aged 25–44 who lived alone between 1990 and 1995 and they
The percentage of people aged 25–44 who lived alone increased abruptly between 1990 and 1995 and has
There has been an abrupt increase in the percentage of people aged 25–44 living alone between 1990 and 1995 and it
Between 1990 and 1995, there was an abrupt increase in the percentage of people aged 25–44 who lived alone which
Step 1: Read the Original Sentence Carefully, Looking for Errors
The original sentence sounds correct and exhibits parallel structure: “increased abruptly … and continued to rise more slowly …” But you should check out the other choices to see whether you may have missed something.
Step 2: Scan and Group the Answer Choices
Note that (B), (D), and (E) contain “there was” or “there has been.” This type of construction is rarely correct on the GMAT. Unless the other choices commit grievous errors, be prepared to eliminate these.
Step 3: Eliminate Choices Until Only One Remains
You can eliminate (B), (D), and (E) because “there” constructions (“there was,” “there has been,” etc.) should generally be avoided; these constructions are wordy and often turn verbs into passive voice. (C) is wrong here because the present perfect tense, “has continued,” indicates that the action began in the past but continues into the present. Because the sentence is discussing what happened between 1995 and 2000, the simple past tense is correct. Therefore, (A) is in fact correct.
44. (A)
The macabre nature of Macbeth, together with the widespread belief that real-life tragedies have accompanied many productions, has made the name of the play so dreaded that not even the least superstitious members of most casts dare utter it.
has made the name of the play so dreaded that not even the least superstitious members of most casts dare
have made the name of the play so dreaded, even the least superstitious members of most casts will not dare to
have made the play’s name sufficiently dreaded, so that even the less superstitious members of most casts will not dare to
have made the name of the play sufficiently dreaded, so that not even the least superstitious members of most casts dare
has made the name of the play is dreaded, so that even the least superstitious members of most casts dare not
Step 1: Read the Original Sentence Carefully, Looking for Errors
You should learn to ignore long phrases between commas when you first read the sentence. If you do that you’ll see that the subject of the sentence, “nature,” is singular, so the verb should be singular too, and “has made” fits that bill.
Step 2: Scan and Group the Answer Choices
One split here is between “has” in (A) and (E) and “have” in (B), (C), and (D). Use that split to begin eliminating answer choices.
Step 3: Eliminate Choices Until Only One Remains
Because “has made” is the correct verb for a singular subject, eliminate (B), (C), and (D). Eliminate (E) because it contains an extra verb, “is,” that makes the sentence unclear. (A) is the only choice left, which means the sentence must be correct as written. Note that (A) also contains the correct construction “so dreaded … that.”
45. (C)
The university’s board of trustees, being worried over declining student enrollments and their failing to secure additional funding from the state, has formed a committee to determine what cuts need to be made to staff and programs.
being worried over declining student enrollments and their failing
worrying over declining student enrollments and also the failure
worried about declining student enrollments and the failure
in that they are worried about the decline in student enrollments and the failure
because of its worry concerning the decline of student enrollments and, as well as concerning the failure
Step 1: Read the Original Sentence Carefully, Looking for Errors
The sentence as written is wrong because the underlined portion begins with the suspicious and unnecessary “being,” which is rarely correct on the GMAT. Another thing to notice is that the underlined portion contains a two-part list, so you should check for parallel structure. As written, the board is worried over (1) “declining student enrollments” and (2) “their failing to secure…” This is not parallel. Finally, notice the original sentence’s improper use of the pronoun “their.” The antecedent is “board,” and the members of the board are acting as a unit, so the correct pronoun would be “its.” (And notice, by the way, that the non-underlined part of the sentence includes the singular verb “has.” This alerts us that the plural pronoun “their” is not correct here.)
Step 2: Scan and Group the Answer Choices
All four of the viable answer choices, (B), (C), (D), and (E), correct the pronoun error and the parallel structure error—the original sentence’s “their failing” is replaced in each of these choices by “the failure”—so we can turn our focus to the first part of each choice. The options are “worrying,” “worried,” “in that they are worried,” and “because of its worry.” (D)’s “in that they are worried” is verbose and introduces another it/they pronoun error, so we can eliminate it right away. Now we must decide which of the three remaining choices, (B), (C), and (E), is the most economical and idiomatic.
Step 3: Eliminate Choices Until Only One Remains
(E)’s phrase “because of its worry” is needlessly wordy, so we can strike it out. (B)’s phrase “worrying over” is not idiomatic. Notice, too, that (B) adds a superfluous word: “also.” (C) contains the correct idiom “worried about” and does not add any excess verbiage. (C) is the correct answer.
46. (E)
The United States’ trade deficit with China rose in 2003 to $123 billion, which is 17 percent more than the previous year and more than 10 times the U.S.-China trade deficit in 1998.
which is 17 percent more than the previous year
which is 17 percent higher than it was the previous year
17 percent higher than the previous year’s figure was
an amount that is 17 percent more than the previous year was
an amount that is 17 percent higher than the previous year’s figure
Step 1: Read the Original Sentence Carefully, Looking for Errors
Because the relative pronoun “which” refers to the noun that comes immediately before it, this means the underlined segment illogically compares a sum of money, “$123 billion,” to “the previous year.” The correct answer must compare a sum and a sum, not a sum and a year.
Step 2: Scan and Group the Answer Choices
(A) and (B) both start with “which.” (C) leaves out “which,” while (D) and (E) substitute “an amount that” for “which.”
Step 3: Eliminate Choices Until Only One Remains
We can eliminate (A) because of its illogical comparison. (B) also screws up the comparison, thanks to its careless use of the slippery pronoun “it.” Because “which” refers to “123 billion,” (B) seems to say, nonsensically, that $123 billion is 17 percent higher than $123 billion was the previous year. (D) also fails to fix the illogical comparison; it compares “an amount” to “the previous year.” That leaves only (C) and (E). Both of these choices correctly compare two amounts of money. However, (C) adds “was” at the end, which is unnecessary and not parallel with the last part of the sentence, which does not use the verb. (E), which introduces no new errors, is the correct answer.
47. (D)
The two-party political system is one where the electorate gives its votes largely to only two major parties and where one or the other party can usually win a majority in the legislature.
where the electorate gives its votes largely to only two major parties and where
in which the electorate largely gives only its votes to two major parties and where
where the electorate gives largely its votes to only two major parties and in which
in which the electorate gives its votes largely to only two major parties and in which
in which the electorate largely gives only its votes to two major parties and in which
Step 1: Read the Original Sentence Carefully, Looking for Errors
The underlined part of this sentence contains two incorrect uses of “where.” On the GMAT, “where” is used correctly only if it refers to location. (For instance, “this is the café where we had lunch last week.”) In the original sentence, “where” refers, both times, to the “two-party political system.” A system is not a location, so we have an error, and we know that (A) is incorrect.
Step 2: Scan and Group the Answer Choices
Of the remaining choices, (B) substitutes “in which” for the first “where,” but it retains the second “where.” (C) keeps the first “where” and substitutes “in which” for the second. (D) and (E) substitute “in which” for both uses of “where” in the original sentence.
Step 3: Eliminate Choices Until Only One Remains
We can eliminate (B) and (C) because each still contains an incorrect usage of “where” to refer to something other than a location. That leaves (D) and (E) as the remaining contestants. The difference between them is that (E) moves the placement of “only,” so instead of “gives its votes largely to only two major parties,” the sentence now reads, “gives only its votes to two major parties.” It’s a small change, but it alters the meaning of the sentence dramatically. (E) makes it sound as if the electorate had other things besides its votes to give to the parties, which is not a meaning implied by the original sentence. We’re left with (D) as the correct answer, since it fixes the original mistake without introducing any new errors and without changing the meaning of the original sentence.
48. (C)
Most financial advisers recommend that stock portfolios should be reviewed at least once a year, if only to make sure that changes do not need to be made.
recommend that stock portfolios should be reviewed
recommend you to review your stock portfolio
recommend that stock portfolios be reviewed
are recommending that stock portfolios are reviewed
have a recommendation to review stock portfolios
Step 1: Read the Original Sentence Carefully, Looking for Errors
Underlined verbs demand our attention. The subject of “recommend” is “advisers.” Subject and verb are both plural, so there is no subject-verb agreement error. However, verbs describing orders and recommendations must be followed by “that,” a new subject, and another verb in the infinitive, but without the “to.” In this sentence, the phrase that follows “recommend” contains the extra, unnecessary word “should.” Choice (A) will not be the correct answer.
Step 2: Scan and Group the Answer Choices
Of the remaining choices, (C) and (D) preserve “that,” while (B) and (E) leave it out.
Step 3: Eliminate Choices Until Only One Remains
We can eliminate (B) and (E) because orders and recommendations must be followed by “that.” Moreover, “have a recommendation” in (E) is wordy and awkward. (D) uses “that,” but it introduces a new error: instead of “be,” it uses “are.” Only (C) remains and must be correct. Indeed, (C) has exactly the right grammatical structure for a recommendation: “that … be reviewed.” Choice (C) is the correct answer.
49. (D)
The United States would achieve a 10 percent reduction in gasoline consumption, if Congress will raise fuel economy standards to 31.3 miles per gallon for passenger cars and to 24.5 mpg for light trucks.
would achieve a 10 percent reduction in gasoline consumption, if Congress will raise
will achieve a 10 percent reduction in gasoline consumption, if Congress were to raise
will have achieved a 10 percent reduction in gasoline consumption, if Congress will raise
would achieve a 10 percent reduction in gasoline consumption, if Congress were to raise
would achieve a 10 percent reduction in gasoline consumption, if Congress were raising
Step 1: Read the Original Sentence Carefully, Looking for Errors
The word “if” in the underlined part of this sentence signals a hypothetical situation, so we need the subjunctive mood; we must use “were” and “would.” “Would achieve” is correct, but “will raise” is not. We can rule out (A).
Step 2: Scan and Group the Answer Choices
(B) and (C) start with “will.” (D) and (E) start with “would.”
Step 3: Eliminate Choices Until Only One Remains
(B) is incorrect because “will achieve” is ordinary future tense, not subjunctive. Similarly, (C) is incorrect because “will have achieved” is also not subjunctive. (D) and (E) both use “would achieve,” which is the proper subjunctive mood. The difference between them comes at the end. (D) uses “if Congress were to raise,” while (E) has “if Congress were raising.” The final phrase in (E) sounds a bit odd and is in fact grammatically incorrect. The proper subjunctive construction calls for the verb that follows “were” to be in the infinitive, not the –ing form. Choice (E) is thus incorrect, and (D), which contains the proper “were”-plus-infinitive construction, is the correct answer.
50. (C)
Because the moon’s distance from Earth varies, so the gravitational pull between the two bodies differ, as do the size and times of oceanic tides.
so the gravitational pull between the two bodies differ
so the gravitational pull between the two bodies differs
the gravitational pull between the two bodies differs
therefore the gravitational pull between the two bodies differ
therefore the gravitational pull between the two bodies differs
Step 1: Read the Original Sentence Carefully, Looking for Errors
Two clauses need to be connected by one, and only one, conjunction. This sentence connects its introductory clause to its main clause with two conjunctions, “because” and “so.” That’s one conjunction too many. Moreover, this sentence contains a subject-verb agreement error. The subject of “differ” is “gravitational pull,” which is singular. The verb should be “differs.” The correct answer will fix both these mistakes without introducing any new ones.
Step 2: Scan and Group the Answer Choices
The choices group naturally with respect to the conjunction. (A) and (B) retain “so.” (D) and (E) substitute “therefore,” while (C) eliminates the conjunction entirely.
Step 3: Eliminate Choices Until Only One Remains
We’ve already determined that “Because” at the beginning of the sentence is the only conjunction we need to connect the first and second clauses of this sentence. (A), (B), (D), and (E) all have an extra conjunction and are therefore all incorrect. Only (C) eliminates the superfluous conjunction. It also changes “differ” to “differs,” so it fixes both mistakes and is the correct answer.
51. (A)
With a boiling temperature of –195.8 degrees Celsius, nitrogen composes approximately 78 percent of the volume of the atmosphere.
nitrogen composes approximately 78 percent of the volume of the atmosphere
nitrogen is composing the volume of 78 percent of the atmosphere
the atmosphere is approximately 78 percent nitrogen, as measured by volume
nitrogen is composed of 78 percent of the atmosphere’s volume
the atmosphere is composed, in terms of volume, of 78 percent nitrogen
Step 1: Read the Original Sentence Carefully, Looking for Errors
The introductory phrase followed by a comma indicates that this question is testing modifiers. So you need to be sure that the subject being modified is in the right position relative to the modifier. Since the modifier in this case is “With a boiling temperature of … ,” the subject being modified must be something that has a boiling point, and it must come right after the comma. “Nitrogen” has a boiling point and is the logical subject of the modifier. The rest of the underlined portion is correct as well, but scan the choices just in case another choice somehow expresses this idea more concisely.
Step 2: Scan and Group the Answer Choices
(A), (B), and (D) begin with “nitrogen,” while (C) and (E) begin with “the atmosphere.”
Step 3: Eliminate Choices Until Only One Remains
Based on the opening modifier, eliminate (C) and (E), since it doesn’t make sense for the atmosphere to have a boiling point. (B) has a strange construction (“is composing the volume of”) and should be eliminated on that basis. Also eliminate (D), which makes it sound as though nitrogen itself contains 78 percent of the atmosphere’s volume (how is that possible?), when the sentence intends to communicate that 78 percent of the volume of the atmosphere is made up of nitrogen. Since no other choice expresses the main idea of the sentence more clearly and more correctly than the original, (A) is correct.
52. (D)
At more than 800 pages in length, the novelist’s life stands revealed in the new biography, which sets the standard for research.
novelist’s life stands revealed in the new biography, which sets the standard for research
new biography is setting standards of research on the novelist’s life
standard for research has been set by a new biography on the novelist
new biography about the novelist sets the standard for research
novelist’s life is revealed in a standard-setting new biography
Step 1: Read the Original Sentence Carefully, Looking for Errors
The opening phrase of this sentence is a modifier, and modifiers on the GMAT always need to be placed as close to what they modify as possible. When you see such opening phrases, ask yourself, “What in the sentence is this phrase describing?” Here, the only thing in the sentence that could be more than 800 pages in length is “the new biography,” so that should come right after the comma.
Step 2: Scan and Group the Answer Choices
In GMAT sentences that open with a modifying phrase set off by a comma, what comes after the comma is crucial. (A) and (E) begin with the “novelist’s life,” (B) and (D) begin with the “new biography,” and (C) begins with the “standard for research …”
Step 3: Eliminate Choices Until Only One Remains
Get rid of (A), (C), and (E), because none of them has the correct subject for the modifier. Only (B) and (D) use the correct subject after the phrase. But the verb tense in (B) changes the original sense of the sentence (“is setting” would indicate that the process of setting the standard hasn’t yet been completed, which doesn’t make sense). Only (D) has the correct subject and the correct tense.
53. (D)
Because women buy approximately 80 percent of ties sold in the United States, they are often displayed near perfume or women’s clothing departments.
they are often displayed
ties are often being displayed
the displaying of ties is often
ties are often displayed
they often can be found
Step 1: Read the Original Sentence Carefully, Looking for Errors
As you read more and more GMAT Sentence Correction questions, you’ll develop a keen sense of the words that are the biggest “troublemakers.” One of them is the pronoun “they,” a pronoun that is often used ambiguously in everyday speech but must have a clear antecedent on the GMAT. The problem with this sentence is that it’s unclear whether the pronoun “they” refers to “women” or “ties.” Of course, logic dictates that it must refer to the ties, but the structure of the sentence doesn’t make this explicit.
Step 2: Scan and Group the Answer Choices
(A) and (E) use “they.” (B), (C), and (D) don’t contain pronouns, but these choices contain largely the same words in very different arrangements.
Step 3: Eliminate Choices Until Only One Remains
Once you spot a pronoun ambiguity error, keep in mind that the right answer may very well dispense with pronouns altogether. Because “they” is being used so ambiguously here, eliminate (A) and (E). Eliminate (C) because of its awkward construction (“the displaying of ties” turns the verb “display” into a gerund, which the GMAT is not fond of). (B) is out because it needlessly uses another problematic GMAT word—“being.” Only (D) clears up the ambiguous pronoun in a correct, concise way.
54. (C)
Although the British were responsible for the early European settlement of both Australia and the United States, Australia is having much closer political and cultural links with Britain than the United States is having.
of both Australia and the United States, Australia is having much closer political and cultural links with Britain than the United States is having
in both Australia and the United States, they are closer in their political and cultural links in Australia than in the United States
of both nations, Australia has much closer political and cultural links to Britain than the United States has
in both nations, Australia is politically and culturally linked to Britain in a much closer fashion than the United States
of both Australia and the United States, they have much closer political and cultural links to it than the United States has
Step 1: Read the Original Sentence Carefully, Looking for Errors
Note the strange verb tense—present progressive—used in the underlined portion. The –ing tense is usually wrong on the GMAT and is certainly not correct if the intention is to express a permanent state of affairs, as is the case here. Also, in most cases, the present progressive can be ditched in favor of the more concise present tense. (Why use “is having” when you can use “has”?) The only time that the GMAT tends to use the progressive tense in the correct answer is when doing so changes a misplaced modifier to a verb. (Example: “The dog walked down the street, which was carrying a stick” should actually be “the dog walked down the street, carrying a stick.”) Here, there’s no good reason for the verb construction “is having,” so it’s incorrect.
Step 2: Scan and Group the Answer Choices
As you scan, you might notice that the sentence is also testing an idiom issue. (A), (C), and (E) all begin with “of,” while (B) and (D) begin with “in.” This difference forces you to decide which one is correct, even if you hadn’t thought of it while reading the original sentence.
Step 3: Eliminate Choices Until Only One Remains
The correct idiom is “settlement of,” not “settlement in,” so eliminate (B) and (D). The odd choice of verb tense in the original sentence disqualifies (A). Finally, (E) can be eliminated because it includes the pronouns “it” and “they,” which make the sentence ambiguous. Only (C) corrects the verb tense and does so in a clear, concise manner.
55. (E)
Each of the major setbacks the Germans suffered in June 1944—the fall of Rome, the collapse of Army Group Center on the Eastern Front, and the breaching of the Atlantic Wall at Normandy—were powerful shocks signaling the inability of the German war machine to compete on multiple fronts.
Each of the major setbacks the Germans suffered in June 1944—the fall of Rome, the collapse of Army Group Center on the Eastern Front, and the breaching of the Atlantic Wall at Normandy—were powerful shocks
The fall of Rome, the collapse of Army Group Center on the Eastern Front, and the breaching of the Atlantic Wall at Normandy—each of them major setbacks the Germans suffered in June 1944—were powerful shocks
Powerful shocks—the fall of Rome, the collapse of Army Group Center on the Eastern Front, and the breaching of the Atlantic Wall at Normandy—each a major setback for the Germans in June 1944, was such
The major setbacks suffered by the Germans in June 1944—the fall of Rome, the collapse of Army Group Center on the Eastern Front, and the breaching of the Atlantic Wall at Normandy—each were powerful shocks
Each of the major setbacks the Germans suffered in June 1944—the fall of Rome, the collapse of Army Group Center on the Eastern Front, and the breach of the Atlantic Wall at Normandy—was a powerful shock
Step 1: Read the Original Sentence Carefully, Looking for Errors
Note that this question is testing two classic GMAT Sentence Correction concepts. The first, parallel structure, was probably easier to spot since the sentence uses dashes to set off a list of the setbacks that the Germans faced. Those elements—(1) “the fall,” (2) “the collapse,” and (3) “the breaching”—are not parallel, since “breaching” is a gerund. Also, the use of a lengthy phrase between dashes should signal this sentence’s attempt to disguise a subject-verb agreement error. The subject of the sentence is not “setbacks” but “each,” which is singular, so the verb “were” doesn’t agree in number with this subject. (The subject of a sentence will never appear in a prepositional phrase, as “setbacks” does.)
Step 2: Scan and Group the Answer Choices
Because of the verb issue, see if you can scan the choices quickly to note the verb each uses. (B) and (D), like the original sentence, use “were,” so unless one of these choices changes the subject of the sentence to something plural, these can probably be eliminated.
Step 3: Eliminate Choices Until Only One Remains
Get rid of (A) right away because of both errors present there. (B), (C), and (D) all fail to correct the parallelism error, so they can all be eliminated. Only (E) has parallel structure and uses the singular verb “was.” (Note that (D) also continues to use “each … were” and (C) is incredibly confusing and poorly constructed. Often the GMAT will put multiple errors in an underlined portion to give you more than one opportunity to eliminate wrong answers.) Thus, choice (E) is the correct answer.
56. (C)
Adult pelicans store fish in a deep, expandable pouch below the lower mandible, of which the young feed.
of which the young feed
being for the feeding of the young
from which the young feed
that it feeds the young from
young feed from
Step 1: Read the Original Sentence Carefully, Looking for Errors
This underlined portion is quite short, but a quick reading indicates that this question is testing idiomatic usage. Young pelicans do not feed “of” something, as the original sentence says. They feed “from” something.
Step 2: Scan and Group the Answer Choices
As you scan the choices, try to spot constructions and individual words that are not likely to be correct, such as “being” in (B).
Step 3: Eliminate Choices Until Only One Remains
To “feed of” something is incorrect, so (A) is out. Eliminate (B) because it is way too wordy and uses the disfavored “being” construction. Eliminate (D) and (E) because “from” is left dangling at the end of the sentence, and it is considered poor form on the GMAT to end a sentence with a preposition. Furthermore, (D) uses the singular pronoun “it” to stand in for plural “pelicans,” and (E) hardly makes sense as written; it merely tacks on a separate clause with no conjunction to connect it to the rest of the sentence. Only (C) is both idiomatically and grammatically correct.
57. (B)
Anthony Trollope, one of the most famous English novelists of the nineteenth century, also wrote travel books, some based on his undertaking of journeys while an employee of the postal service.
wrote travel books, some based on his undertaking of journeys
wrote travel books, some based on journeys he undertook
undertook journeys, having based travel books on them
based travel books on journeys undertaken
wrote travel books, some based on journeys which he was to undertake
Step 1: Read the Original Sentence Carefully, Looking for Errors
The beginning of the underlined portion contains the verb “wrote,” which is correct here given the appropriateness of the simple past tense. The problem with this sentence is one of usage and style. As we have seen, the GMAT disfavors –ing constructions. The phrase “based on his undertaking” is needlessly awkward.
Step 2: Scan and Group the Answer Choices
There are no discernible splits here. Since “wrote” is correct, though, be suspicious of (C) and (D), which change the verb to something different.
Step 3: Eliminate Choices Until Only One Remains
Because of the awkward construction in the original sentence, eliminate (A). Eliminate (C), which makes it sound as though Trollope based the travel books on the journeys before he took the journeys. Similarly, eliminate (E) because the travel books were based on journeys he had already undertaken, not on journeys he would undertake in the future. Eliminate (D), which makes it unclear who is actually undertaking the journeys. Only (B) keeps the meaning of the sentence intact while adhering to GMAT style. Thus, choice (B) is the correct answer.
58. (B)
A federal government survey taken in 1997 showed that during the 12 months preceding the survey, 36 percent of the United States population had tried marijuana, cocaine, or other illicit drugs, 71 percent of the population were smoking cigarettes, and 82 percent tried alcoholic beverages.
were smoking cigarettes, and 82 percent
had smoked cigarettes, and 82 percent had
smoked cigarettes, and that 82 percent
smoked cigarettes, and 82 percent had
will be smoking cigarettes, and 82 percent
Step 1: Read the Original Sentence Carefully, Looking for Errors
This question is testing parallel structure: similar elements must be in a similar form. Here you have three statistics being compared, and the one that is not underlined reads “36 percent … had …” Therefore, the other two statistics must be in the same form.
Step 2: Scan and Group the Answer Choices
Note the variety of verb tenses: (A) uses “were smoking,” (B) uses “had smoked,” (C) and (D) use “smoked,” and (E) uses “will be smoking.”
Step 3: Eliminate Choices Until Only One Remains
“Had smoked” is correct for two reasons. First of all, it’s the past perfect tense, which should always be used to describe actions occurring before other actions in the past. Here, before the survey was taken in 1997, people had used drugs, cigarettes, and alcohol. Second, “had smoked” is parallel to the elements of the sentence that are not underlined and that therefore cannot be changed. Eliminate (A), (C), (D), and (E) because they use tenses that don’t make sense given the sequence of events in the sentence and because they do not put similar elements in a similar form. Only choice (B) corrects both problems.
59. (C)
Like Art Nouveau jewelry designers, the Art Deco movement used art materials suitable for expressing the new stylistic language of the 1920s, one dominated by an interplay of geometric forms.
Art Deco movement used art materials suitable for expressing
Art Deco movement was expressed through the use of materials suitable for the
artists of the Art Deco movement used materials suitable for expressing
Art Deco movement’s materials were used by artists to suitably express
artists of the Art Deco movement also used suitable materials for the expression of
Step 1: Read the Original Sentence Carefully, Looking for Errors
As soon as you see “like” at the beginning of the sentence, know that a comparison is being made. The GMAT is fond of testing your ability to distinguish between logical and illogical comparisons. As written, the sentence compares “Art Nouveau jewelry designers” to “the Art Deco movement,” which is an illogical comparison. The beginning of the underlined portion will need to be changed to something more comparable to “jewelry designers.”
Step 2: Scan and Group the Answer Choices
(A) and (B) begin with “Art Deco movement,” (C) and (E) begin with “artists of the Art Deco movement,” and (D) begins with “Art Deco movement’s materials.” Only one of these beginnings will be correct.
Step 3: Eliminate Choices Until Only One Remains
“Artists of the Art Deco movement” is correct, since “artists” can properly be compared to “jewelry designers” in the beginning of the sentence. Therefore, eliminate (A), (B), and (D). Then eliminate (E) because “also” here is redundant (since the sentence already begins with “like”) and because it is less concise than (C), which is the only choice that remains.
60. (C)
Amtrak, a government-owned corporation, schedules passenger rail service, payments to privately owned firms to run trains, and bears all administrative costs, such as those incurred by the purchase of new equipment and by the sale of tickets.
payments to privately owned firms to run trains, and bears all administrative costs, such as
pays privately owned firms to run trains, and is bearing all administrative costs, such as
pays privately owned firms to run trains, and bears all administrative costs, such as
pays privately owned firms to run trains, and bears all administrative costs, being inclusive of
pays privately owned firms to run trains, and the bearer of all costs of administration, for example
Step 1: Read the Original Sentence Carefully, Looking for Errors
The sentence presents a list of Amtrak’s responsibilities. The first one is a verb in the present tense, “schedules,” and this verb isn’t underlined, so the other two elements in the list must be parallel to this form. As it stands, “payments” is not parallel with “schedules” and “bears.”
Step 2: Scan and Group the Answer Choices
Every choice except for (A) uses “pays.” Furthermore, (A), (C), and (D) use “bears,” while (B) uses “is bearing” and (E) uses “the bearer.”
Step 3: Eliminate Choices Until Only One Remains
Because of the original parallelism error, eliminate (A). Eliminate (B) and (E) because they change the verb tense in the second part of the clause from “bears” to a non-parallel construction. Finally, eliminate (D) because “being inclusive of” is wordy and awkward and contains the oft-incorrect “being.” Choice (C) has all elements parallel and, thus, is the correct answer.
61. (A)
In feudal Korea, cattle were traditionally used as beasts of burden, rather than as a food source, and fish were the primary source of protein in the typical diet.
cattle were traditionally used as beasts of burden, rather than as a food source, and fish were
cattle were being used as beasts of burden, rather than as a food source, and fish were providing
cattle were traditionally used as beasts of burden, rather than a food source, with fish being
the traditional use of cattle was as beasts of burden, rather than for the provision of food, and fish were
cattle were traditionally used for beasts of burden, rather than for a food source, and fish were
Step 1: Read the Original Sentence Carefully, Looking for Errors
When reading the sentence, you may have noticed that the sentence uses passive voice in the beginning of the underlined portion (“cattle were … used”). Normally, the GMAT does not prefer to use passive voice because it is wordier and takes the focus off the person/thing completing the action of the sentence. In order to select a better alternative, though, you’ll need to pinpoint the specific people who used the cattle as beasts of burden.
Step 2: Scan and Group the Answer Choices
(A), (B), (C), and (E) all continue to use passive voice in the beginning of the underlined portion. (D) avoids this in favor of the wordy “the traditional use of cattle was as …” None of the sentences actually make clear who is completing the action described. Without a clear way to eliminate using passive voice to group the answer choices, we must evaluate them one by one, eliminating those that contain identifiable flaws.
Step 3: Eliminate Choices Until Only One Remains
Because the sentence discusses traditional uses, all verbs should be in the past tense. Eliminate (B) and (C) because they use the incorrect –ing verb tense in the second part of the sentence; (B) uses “were providing” at the end, which is awkward, and (C) contains “being,” which is rarely correct on the GMAT. Eliminate (D) because it is overly wordy; whenever there is some other answer choice that expresses the same idea just as correctly but with fewer words, select that answer. Eliminate (E) because it incorrectly substitutes “used for” for “used as.” The sentence is correct as written, making (A) the answer.
62. (E)
The term “support staff” is often used to describe employees that perform lower-level tasks but also are providing essential administrative duties for executive-level managers.
that perform lower-level tasks but also are providing
performing lower-level tasks but also providing
who perform lower-level tasks, but are also providing
that performs lower-level tasks but also provide
who perform lower-level tasks but also provide
Step 1: Read the Original Sentence Carefully, Looking for Errors
Always pay close attention to the beginnings of underlined portions—these beginnings are not chosen at random. The underlined portion of this sentence begins with “that.” Looking at what comes before, you should notice that the previous word, “employees,” requires the pronoun “who” on the GMAT, since the noun “employees” refers to people. Also, the verb tenses of the elements that are being compared should be the same, and here, “perform” doesn’t match “are providing.”
Step 2: Scan and Group the Answer Choices
(A) and (D) begin with “that.” (C) and (E) begin with “who.” (B) gets rid of the pronoun altogether and begins with “performing.”
Step 3: Eliminate Choices Until Only One Remains
The modifier should begin with “who” rather than “that,” so (A) and (D) should be eliminated. Get rid of (B) because there is no need to use the progressive verb tense (“performing” and “providing”) when the simple present tense conveys the idea clearly. Eliminate (C) because the verb forms are not parallel. Choice (E) is correct because it uses the pronoun “who” and the verbs are parallel, both using the simple present tense.
63. (A)
National Women’s History Month began as a single week and as a local event when, in 1978, Sonoma County, California, sponsored a women’s history week to promote the teaching of women’s history—a neglected subject in elementary and high school curricula at that time.
began as a single week and as a local event when, in 1978, Sonoma County, California, sponsored
began as a single week local event sponsoring, in Sonoma County, California,
begins with a single week in 1978 sponsoring the local Sonoma County, California of
will have began in 1978 as a single week local event when Sonoma County, California was in sponsorship of
was begun in 1978 when, in Sonoma County, California, there was to be a sponsorship of
Step 1: Read the Original Sentence Carefully, Looking for Errors
From the beginning of the underlined portion, you can tell that this question tests verb tenses and clarity. Since the sentence discusses something that happened in 1978, all verbs should be in the past tense, and the structure of the sentence should reflect the facts that National Women’s History Month was both a single week and a local event and that it was sponsored by Sonoma County. All of that information is clearly reflected in the question stem, so it is most likely correct. However, it is important to check all the answer choices to make sure that none of them are more concise, especially in such a long sentence.
Step 2: Scan and Group the Answer Choices
The most obvious difference in the answer choices is in their beginnings. (A) and (B) use “began,” while the remaining choices use a variety of tenses ranging from present in (C) to future perfect in (D) to a passive construction in (E).
Step 3: Eliminate Choices Until Only One Remains
Eliminate (B) because it does not make it clear that Sonoma County sponsored the event. Eliminate (C) both because it uses the incorrect verb tense and because it suggests that the single week sponsors Sonoma County. Eliminate (D) because it uses the incorrect verb tense. Eliminate (E) because “there was to be a sponsorship” is awkward and wordy. This leaves choice (A) as the correct answer.
64. (D)
People with schizophrenia experience miscarriages at a higher rate of frequency than people without schizophrenia, whose overactive immune systems tend to be indiscriminately rejecting the foreign DNA of the fetus.
People with schizophrenia experience miscarriages at a higher rate of frequency than people without schizophrenia, whose overactive immune systems tend to be indiscriminately rejecting
Miscarriages are more common among people with schizophrenia than among people without schizophrenia, being that their overactive immune systems tend to indiscriminately reject
Miscarriages are more common between people with schizophrenia than they are between people without schizophrenia because the overactive immune systems of people with schizophrenia tend to be indiscriminately rejecting
The miscarriage rate of people with schizophrenia is higher than the rate for people without schizophrenia because the former’s overactive immune systems tend to reject
People with schizophrenia have more common miscarriages than do people without schizophrenia, because their immune systems tend to indiscriminately reject
Step 1: Read the Original Sentence Carefully, Looking for Errors
The original sentence uses the pronoun “whose” next to “people without schizophrenia,” but read closely: are they the ones who have the overactive immune systems? No, that would be the people with schizophrenia, because their overactive immune systems are causing them to miscarry more frequently. Therefore, the original sentence doesn’t make sense and will need to be changed.
Step 2: Scan and Group the Answer Choices
It’s hard to find splits here, and the choices are long, but watch for whether the answer choices will continue to use “whose” and where they will place that pronoun. There is one noticeable split: (A) and (C) end with “rejecting” while (B), (D), and (E) all end with “reject.”
Step 3: Eliminate Choices Until Only One Remains
The sentence as written does not properly express its meaning, so (A) is out. Eliminate (C) because of its misuse of the word “between” and its use of the wordy “tend to be indiscriminately rejecting.” Eliminate (B) because of the awkward “being that” construction and its ambiguous use of the pronoun “their.” Eliminate (E) because the pronoun “their” is ambiguous and could refer to either group. Only (D) expresses the sentence’s meaning clearly and concisely.
65. (A)
Working with musician Stevie Wonder as a musical adviser, Ray Kurzweil developed what is now the preferred medium for creating nearly all music for today’s commercial albums, films, and television: a computer-based instrument capable of reproducing musical sounds of a quality once thought to be possible only on grand pianos and other acoustic instruments.
reproducing musical sounds of a quality once thought to be
reproducing musical sounds with a quality as were once thought to be
reproducing in musical sounds a quality that was once thought to be
reproduction of musical sounds whose quality is the same as what were once thought to be
reproduction of musical sounds at a quality once thought of as
Step 1: Read the Original Sentence Carefully, Looking for Errors
This is a long sentence, but the colon indicates that what comes after the colon is simply going to explain more about what comes before the colon, so let’s just focus on what comes after the colon, since that’s where the underlined portion is. The original sentence may have sounded just fine to you. Before selecting (A), though, make sure none of the other choices expresses the idea more clearly and concisely.
Step 2: Scan and Group the Answer Choices
(A) and (B) begin with “reproducing musical sounds,” (C) begins with “reproducing in musical sounds,” and (D) and (E) begin with “reproduction of musical sounds.” It appears that this sentence is testing idiomatic constructions.
Step 3: Eliminate Choices Until Only One Remains
The original sentence is correct as written, making (A) the correct answer. It contains the correct constructions: “capable of reproducing … sounds of a quality once thought to be possible …” In (B), “with a quality” changes the sentence to mean that the instrument possesses the “quality,” when it’s the quality of the sounds that’s discussed here; furthermore, “as were once” is unidiomatic in the sentence. (C) unnecessarily introduces “in” and “that was.” Extraneous words are a sign of an incorrect answer in GMAT Sentence Correction questions. (D) and (E) introduce the awkward construction “capable of reproduction of.” (D) is also extremely wordy, and (E) also includes the unidiomatic “at a quality” and “once thought of as.”
66. (A)
Twenty years ago, only 6 percent of students at Dunmore College who intended to major in chemistry were women; today that figure is at least as high as 40 percent.
at least as high as 40 percent
higher than at least 40 percent
higher by 40 percent at the least
40 percent high or more
more higher than 40 percent
Step 1: Read the Original Sentence Carefully, Looking for Errors
Comparisons are being tested in this sentence (your hint is the use of the words “at least as high” and the words “higher” and “more” throughout the answer choices). Here you are looking for the construction that makes the comparison as clear and correct as possible. This sentence compares the previous figure, or “6 percent,” with today’s figure, so you want something that is as similar in structure as possible to “6 percent.”
Step 2: Scan and Group the Answer Choices
The comparison sounds good as is, but scan the choices for the ways in which the choices use other comparative words like “higher” (as in (B), (C), and (E)) and “more” (as in (D) and (E)).
Step 3: Eliminate Choices Until Only One Remains
(B) is awkward and redundant; it doesn’t make sense to have both “higher than” and “at least” next to each other. (C) changes the meaning of the sentence by saying “higher by 40 percent,” which means something different from “higher than 40 percent.” (D) introduces the awkward and confusing “40 percent high,” which is hard to decipher. Finally, (E) uses the egregious construction “more higher.” The best choice is (A), “at least as high as 40 percent.”
Now that you’ve learned how to approach Sentence Correction questions on the GMAT, let’s add one more dimension to your understanding of how they work.
Take a moment to try the following question. The next page features performance data from thousands of people who have studied with Kaplan over the decades. Through analyzing this data, we will show you how to approach questions like this one most effectively and how to avoid similarly tempting wrong answer choice types on Test Day.
The European Union announced that cod and mackerel are the only fish that exceeds their new requirements for dioxin level and that they allow fishermen to catch.
exceeds their new requirements for dioxin level and that they allow
exceed its new requirements for dioxin level and that they allow
exceeds its new requirements for dioxin level and that it allows
exceed its new requirements for dioxin level and that it allows
exceed their new requirements for dioxin level and that they allow
Attention to the Right Detail and Pattern Recognition are both essential to success on Sentence Correction questions. You want to check the sentence very carefully for errors, giving special consideration to the recurring patterns that the GMAT testmaker is so fond of. In this sentence, there are two underlined pronouns plus an underlined verb. Both underlined pronouns, “their” and “they,” refer to the European Union, which is a singular noun. These pronouns are therefore incorrect; the sentence should use “its” and “it,” respectively. Only (C) and (D) get both pronouns right, so one of these choices has to be the correct answer. The difference between (C) and (D) is the use of “exceeds” versus “exceed.” Since there are two kinds of fish, “cod and mackerel,” you need the plural verb. Choice (D) is correct.
QUESTION STATISTICS |
2% of test takers choose (A) |
6% of test takers choose (B) |
14% of test takers choose (C) |
56% of test takers choose (D) |
22% of test takers choose (E) |
Sample size = 4,437 |
Note that the two most popular wrong answer choices, (C) and (E), each correct one of the two errors in the sentence but not both. Many test takers will spot an answer choice that fixes the one mistake they were scanning for and think, “Aha—that’s the answer!” and move on without actually reading the answer choice back into the sentence. The testmaker is aware of this tendency. So beware of trap answers that correct just one of two or more errors, as well as trap answers that fix one mistake but introduce another. Always read your choice back into the sentence. It only takes a few seconds and prevents careless mistakes.
To see more questions with answer choice statistics, be sure to review the full-length CATs in your online resources.